Сохранен 507
https://2ch.hk/spc/res/245254.html
Домены arhivach.top и arhivach.site временно не функционируют! Используйте домен ARHIVACH.XYZ.
24 декабря Архивач восстановлен после серьёзной аварии. К сожалению, значительная часть сохранённых изображений и видео была потеряна. Подробности случившегося. Мы призываем всех неравнодушных помочь нам с восстановлением утраченного контента!

Тред тупых вопросов #40

 Аноним 06/01/16 Срд 13:25:01 #1 №245254 
14520759017710.jpg
14520759017721.jpg
14520759017722.jpg
14520759017753.jpg
Тред вопросов о жизни, Вселенной и всем таком.

Спрашиваем то, за что в других местах выдают путёвку в биореактор. Здесь анонимные ученые мирового уровня критически рассмотрят любые гениальные идеи и нарисованные в Paint схемы.

Прошлый тред https://2ch.hk/spc/res/241445.html
Аноним 06/01/16 Срд 14:49:35 #2 №245296 
Какие перспективы развития космоса есть в ближайшие лет 20? Если без всяких внезапных варп-двигателей и прочего.
Аноним 06/01/16 Срд 14:57:11 #3 №245304 
>>245296
Никаких.
Аноним 06/01/16 Срд 15:00:20 #4 №245308 
>>245296
> Если без всяких внезапных варп-двигателей и прочего.
Никаких. Ну может Маск допилит многоразовость, Юла и Роскосмос последуют за ним, цена запуска упадет на порядок, в космос устремится куча бизнеса, который захочет делать гешефт на орбите и на астероидах, космические агентства, пользуясь дешевой доставкой, запилят базы на Луне и Марсе, но на этом месте по законам жанра я должен проснуться на лекции с обосранными штанами.
Аноним 06/01/16 Срд 15:12:47 #5 №245316 
>>245308
У бизнеса вообще есть причина туда соваться, кроме покатушек и телевизионных спутников?
>базы на Луне и Марсе
Нах нужны?
Аноним 06/01/16 Срд 15:20:38 #6 №245323 
>>245316
> У бизнеса вообще есть причина туда соваться, кроме покатушек и телевизионных спутников?
Покатушки не нужны, пока стоимость этого дела не упадет порядка на два. Причины - майнить всякую редкую хуйню на астероидах, например, но тут тоже все очень упирается в стоимость.
> Нах нужны?
Потому что могут.
Аноним 06/01/16 Срд 15:22:46 #7 №245326 
>>245323
А технологии разработки, доставки и всего остального хотя бы в теории есть?
Аноним 06/01/16 Срд 16:52:25 #8 №245348 
>>245326
просто ебошь метеоритным способом, или шаттл like
Аноним 06/01/16 Срд 17:29:28 #9 №245370 
Не совсем релейтед, но похуй. Чому при взрыве термоядерной бомбуэ температура шара достигает порядка 100млн градусов, тогда как в ядре Солнца всего 15млн? Хотя и там, и там водород слипается.
Аноним 06/01/16 Срд 17:58:26 #10 №245375 
>>245370
взрыв термоядерной - не стационарный режим,
который создается другим взрывом.
У солнца равновесный, квазистационарный - температура там могла бы быть и больше, просто наше солнце это желтый карлик - не самое охуенное светило короче, в плане удельной мощности и прочих поражающих воображение моментов, относительно других более крутых в этом план светил конечно, такто оно огого.
Аноним 06/01/16 Срд 20:22:08 #11 №245405 
14521009280500.png
Сохранились еще на Венере те посадочные зонды, что отправлял СССР или она их могла за это время "переварить" своей атмосферой?
Аноним 06/01/16 Срд 20:30:16 #12 №245406 
Поясните про ракетное топливо.
Что гептилом, почему не октановые производные?
Неужели человек не нашёл более горючую смесь чем керосин?
Почему на твёрдом большегрузы не делать?
Что с давлением в баках?
Чём отличаются твёрдое, керосин и водород в плане характеристик?
Что из себя представляет твёрдое? Это жидкое с примесями или сухой порошок?
Аноним 06/01/16 Срд 20:31:36 #13 №245407 
>>245405
Говорят переварила до состояния не лужи, но близкой к этому состоянию.
Аноним 06/01/16 Срд 20:42:05 #14 №245411 
>>245405
Ничего им не стало, кислорода-то нет
Аноним 06/01/16 Срд 20:44:05 #15 №245413 
>>245406
>Что гептилом, почему не октановые производные?
НДМГ удобный - неприхотливый при хранении и гиперголический, не требует поджига при работе с АТ.
Комбинация АТ+НДМГ на 20% плотнее, чем керосин с кислородом.
Ознакомься с таблицей здесь, найдёшь ответы на многие вопросы:
https://en.wikipedia.org/wiki/Liquid_rocket_propellant#Bipropellants
>Неужели человек не нашёл более горючую смесь чем керосин?
НДМГ+АТ
>Почему на твёрдом большегрузы не делать?
Низкий удельный импульс, ergo низкая эффективность, плюс никакой управляемости.
>Что с давлением в баках?
Оно есть. Особенно при вытеснительной системе.
>Чём отличаются твёрдое, керосин и водород в плане характеристик?
См. таблицу выше. Там же рядом ссылки.
>Что из себя представляет твёрдое? Это жидкое с примесями или сухой порошок?
Это твёрдое вещество, всякий люминь с окислителем со связующими смолами. А может и вообще карамель быть, см. "карамельный движок" в гугле, лол.
Аноним 06/01/16 Срд 21:02:17 #16 №245415 
14521033373470.jpg
14521033373581.jpg
>>245406
>Почему на твёрдом большегрузы не делать?
Как это не делать? Пикрилейтед, бустеры тянут до жопы массы.
Собственно они за этим и нужны - обеспечивать сильную тягу. Плотнее топливо = больше тяга, ниже удельный импульс.
Аноним 06/01/16 Срд 21:03:40 #17 №245417 
>>245406
>Чём отличаются твёрдое, керосин и водород в плане характеристик?
Соответственно, и к этому подходит: плотнее топливо = больше тяга/ниже импульс. Водород плохо тянет, но очень эффективно расходуется. Метан тянет лучше, но менее эффективен. Керосин еще менее эффективен и лучше тянет. И т.п.
Аноним 06/01/16 Срд 21:21:54 #18 №245421 
14521045147040.jpg
>Неужели человек не нашёл более горючую смесь чем керосин?
Диоксидифторид + пентаборан
немного кулстори про первое http://blogs.sciencemag.org/pipeline/archives/2010/02/23/things_i_wont_work_with_dioxygen_difluoride Есть еще трехкомпонентные топлива. Но вообще все давно оптимизировано и самые эффективные топливные сочетания давно найдены, дальше только ядерные реакции как источник энергии.

Впрочем эффективность - она разной бывает, например сейчас метан+кислород в моде, т.к. метан не оставляет нагара и позволяет многие вещи в ЖРД сделать проще, или одно-двух компонентные на базе нитрата гидроксиламмония, т.к. высококипящие и могут долго храниться, при этом не так ядовиты как гидразин+АТ (например AF-M315E)
Аноним 06/01/16 Срд 21:25:34 #19 №245422 
>>245405
Электроника сдохла и все. Там только довление и температура в местах посадки, а состав - преимущественно нейтральный СО2 и чуть-чуть азота. Переварить в сульфаты могло бы на высоте, где есть серная кислота - до поверхности она не доживает, разлагается - слишком горячо и довит.

Может быть если на Венере есть активный вулканизм, ветры или пыльные бури - засыпало чем-нибудь, но не более.
Аноним 06/01/16 Срд 21:42:27 #20 №245425 
>>245421
>Диоксидифторид + пентаборан
Удельный 354с, что меньше 382 секунд у воды. Хотя по плотности годно получается. Сопоставимая по плотности комбинация НДМГ+АТ даёт вообще сраные 286с.
Так что да, было бы годно, если бы не фтор фтор кладбище пидор.
>трехкомпонентные
Таки да:
One is a rocket engine which mixes three separate streams of propellants. In the 1960s, Rocketdyne fired an engine using a mixture of liquid lithium, gaseous hydrogen, and liquid fluorine to produce a specific impulse of 542 seconds, likely the highest measured such value for a chemical rocket motor.
Такая-то годнота была бы, да только опять фтор фтор кладбище пидор.
Мне эта табличка очень по нраву, я её чуть выше уже вбрасывал:
https://en.wikipedia.org/wiki/Liquid_rocket_propellant#Bipropellants

Аноним 06/01/16 Срд 21:48:45 #21 №245426 
>>245425
>воды
Тут я очевидно имел в виду топливную пару H2 + O2.
Кстати, тут возник вопрос:
у водорода, как известно, проблемы с долговременным хранением.
А не задумывались ли о заправке йоб ВОДОЙ, электролитическим её разложением перед началом манёвров и употреблением в качестве топлива? И хранить проще вокруг ритэга налейте, например, которым потом расщеплять, чтоб не замерзла и не распидорасила, и по плотности более чем втрое выгоднее, чем хранить порознь.
Я бы предложил и РН заправлять водой, чтобы огромные бочки не городить ради пары тонн водорода, да только быстро передавать столько электричества на борт взлетающей ракеты представляю малореализуемым, равно как и систему подачи гремучего газа.
Его разделять надо при подаче в пихло, кстати?
Аноним 06/01/16 Срд 21:49:46 #22 №245427 
>>245425
>Так что да, было бы годно, если бы
...если бы в этой хуите не горело со взрывом все что принципиально не горит, уже при криогенной температуре, даже ебучий кварцевый песок! линк прочти, заебательская штука по полечению и свойствам, Валентин Петрович ссал бы кипятком; да и вопрос состоял в "горючести", а не в импульсе
Аноним 06/01/16 Срд 21:52:15 #23 №245430 
>>245426
Еб твою мать. Гугли "гидролиз", все сам поймешь
Аноним 06/01/16 Срд 21:53:01 #24 №245433 
>>245426
>гремучего газа
Вот именно, а в двигатели надо жидкий заливать.
Аноним 06/01/16 Срд 21:58:53 #25 №245436 
>>245430
>гидролиз
Но это про разделение всяких солей.
Я про электролиз воды, когда две металлические пластинки в воду, постоянный ток и выделяется гремучая смесь.
Carsandwater на ютьюбе такой станок для своей водородной горелки запилил, которым он никелевый шарик добела калит.

>>245427
Почитаю. Я знал, что фтор - ад, поэтому намеренно не акцентировал на этом внимание, ограничившись лишь
>фтор фтор кладбище пидор
Аноним 06/01/16 Срд 22:00:16 #26 №245437 
>>245254 (OP)
Откуда на Земле взялась вода в таком количестве ? Ведь Земля несколько миллионов лет была океаном из лавы, который постоянно бомбардировали кометы. Вся вода должна была испарится в космос.
Аноним 06/01/16 Срд 22:02:02 #27 №245439 
>>245437
Из комет
Аноним 06/01/16 Срд 22:03:18 #28 №245442 
>>245437
Говорят, что она и испарилась.
А потом кометы ещё накидали, когда уже бурление говн застыло.
Венера сгорела нахуй, марс тоже не смогший в магнитное поле тоже все полимеры проебал, а сраная земляшка обмазалась луной и закрутила магнитное поле, которое как вертухай беглых атомов водорода и молекул воды вертало обратно на сидку.
Аноним 06/01/16 Срд 22:08:53 #29 №245446 
>>245442
Складно базлаешь, чалился где, какой масти будешь?
Аноним 06/01/16 Срд 22:13:11 #30 №245451 
>>245442
А откуда столько воды на Европе и Энцеладе? Там же вообще нет магнитного поля.
Аноним 06/01/16 Срд 22:18:23 #31 №245452 
>>245451
От материнских планет
Аноним 06/01/16 Срд 22:24:05 #32 №245456 
>>245451
Там рядом на минутку гигантские газонюхи. А т.к. этих мужиков посадили недалеко от параши возле газонюхов Питера и сатПрррна, то у них от приливных сил очко жмется и все вертится внутри, даже криометеоризм появляется.
Ну и так далеко от смотрящего по прозвищу Солнце малолетки молекулы воды не стремятся на волю и вполне мерзнут, выпав в осадок.

>>245446
Я с Гришей Галилеичем по прозвищу Телескоп чалился в Атакамской, а потом с Степкой Чайкиным на Кеке мотали срок. Корешами были. Сам я Мишаня Ломайносов, братва в почете.
А ты чьих будешь?
Аноним 06/01/16 Срд 22:43:57 #33 №245467 
14521094371350.png
Что означает аббревиатура MLP на данной картинке?
Попытки загуглить были обречены на провал с известным цветастым результатом, лол.
Аноним 06/01/16 Срд 22:46:13 #34 №245468 
>>245456
А я Плутон Харонович, меня недавно Миша Бурый из планет в карликовые планеты опустил по беспределу
Аноним 06/01/16 Срд 22:52:20 #35 №245474 
>>245467
Mobile Launch Platform?
Аноним 06/01/16 Срд 22:59:22 #36 №245477 
>>245468
Слышал про тебя. Так вас из малолетки много пришло недавно, вон Эридин выёбывался, а за ним и всякие чурки-шкеты вроде Хаумеа и Макемаке. Пока ты один сидел среди мужиков, никто и слова не говорил, а теперь пришлось подопустить зарвавшихся, а то на нарах места не хватает, а под шконарем завались.

Я, кстати, не рассказал, как на кичу попал.
Меня мусор за превышение скорости света тормознул, сказал дуть в трубочку, а я его послал нахуй, не хуесос же я чтобы трубочки в рот брать. На экспертизе показало градус -40 по кельвину и меня в первый раз так и посадили за нарушение принципа причинности в состоянии конденсата Бозе-Эйнштейна. А на Атакамской колонии я и скорефанился с мужиками, и после отсидки на стрелу пришёл во вторую точку Лагранжа, а какая-то крыса нас заложила, и там космический мусор ждал и приняли меня, подкинув пятьдесят грамм антивещества, сел за нарушение законов Кеплера, Ньютона и хранение антиматерии в особо крупных с целью сбыта.
Аноним 06/01/16 Срд 23:04:40 #37 №245480 
>>245477
В голосину!
Аноним 07/01/16 Чтв 07:55:43 #38 №245522 
Почему ядерный распад и синтез останавливается на железе? Я видел ту диаграмму
Аноним 07/01/16 Чтв 09:40:18 #39 №245528 
14521488189100.gif
>>245522
>Почему ядерный распад и синтез останавливается на железе?
Вот такое вот у нас железо во вселенной. У него самые крепкие внутриатомные связи из всех элементов, поэтому распидорасить атом стоит больше энергии, чем он принесет; и синтезировать из него отнимет больше энергии, чем получится в итоге.
Аноним 07/01/16 Чтв 10:02:28 #40 №245533 
14521501489220.jpg
Сколько пройдет лет на Земле если мгновенно телепортироваться на 3 секунды на условно такую же планету как Земля в галактике Андромеда и вернутся? Или тебя просто не будет 3 секунды по земному времени?
Аноним 07/01/16 Чтв 10:06:07 #41 №245535 
>>245533
Если "мгновенно", как ты говоришь - то так и будет. Тебя три секунды тут не было, а потом ты обратно тут.
Если мгновенно для тебя, но с неким подобием соблюдения законов физики, которые пока с неодобрением смотрят на сверхсветовое перемещение, то ты просто полетишь с околосветовой туда/обратно, для тебя пройдёт доля секунды, допустим перегрузками и встречными ионами тебя не распидорашивает, а для земли пройдёт 2,5 мегагода +3 секунды + 2,5 мегагода. Вернешься через 5 с хуем мегалет.
Аноним 07/01/16 Чтв 10:06:56 #42 №245536 
>>245535
>для тебя пройдёт доля секунды
в пути. Доля секунды туда, 3 секунды там, доля секунды обратно.
Релятивистское замедление времени.
Аноним 07/01/16 Чтв 14:41:04 #43 №245613 
Спейсаны, где мне найти профиль полёта ракеты-носителя VEGA?
Аноним 07/01/16 Чтв 14:47:40 #44 №245617 
>>245613
В интернете гугле сайте арианспейс мануале пользователя arianespace.com/wp-content/uploads/2015/09/Vega-Users-Manual_Issue-04_April-2014.pdf
Аноним 07/01/16 Чтв 14:56:10 #45 №245620 
>>245617
>arianespace.com/wp-content/uploads/2015/09/Vega-Users-Manual_Issue-04_April-2014.pdf
Спасибо

другой-анон-который-уже-хотел-написать-тому-анону-что-такого-в-публичном-доступе-нет
Аноним 07/01/16 Чтв 16:32:51 #46 №245646 
А из чего сейчас делают ттрд? Из пороха, как диды?
Аноним 07/01/16 Чтв 17:09:14 #47 №245658 
>>245646
Composites typically consist of a mixture of granules of solid oxidizer (examples: ammonium nitrate, ammonium perchlorate, potassium nitrate) in a polymer binder (binding agent) with flakes or powders of: energetic compounds (examples: RDX, HMX), metallic additives (examples: Aluminium, Beryllium), plasticizers, stabilizers, and/or burn rate modifiers (iron oxide, copper oxide).
https://en.wikipedia.org/wiki/Rocket_propellant#Solid_propellants
Аноним 07/01/16 Чтв 17:11:18 #48 №245659 
>>245646
APCP - основное топливо. Нитрат аммония и в качестве топлива алюминиевый порошок.
Аноним 07/01/16 Чтв 19:25:06 #49 №245693 
Косманы, поясните, были ли американци на луне или это все Кубрик снял?
Аноним 07/01/16 Чтв 19:33:24 #50 №245694 
>>245693
Были
Аноним 07/01/16 Чтв 19:37:59 #51 №245695 
>>245694
А можно поподробнее? почему доводы тех кто утверждает что не были несостоятельны?
Аноним 07/01/16 Чтв 19:38:36 #52 №245696 
>>245693
>>245695
Ты ошибся разделом.
http://2ch.hk/zog/
Аноним 07/01/16 Чтв 19:42:23 #53 №245697 
>>245696
Я бы его лучше на порашу отправил
Аноним 07/01/16 Чтв 19:43:52 #54 №245698 
>>245696
Я не ошибся, я хотел чтобы мне пояснили действительно знающие люди а не всякие доктора филологических наук с рентв.
Аноним 07/01/16 Чтв 19:44:44 #55 №245699 
>>245695
>>245693
Вот тут заговорщиков кормят хуйцами и все их аргументы доказательно и обстоятельно мешают с говном:
http://www.skeptik.net/conspir/moonhoax.htm
Аноним 07/01/16 Чтв 19:50:13 #56 №245702 
>>245699
Спасибо, я удовлетворен
Аноним 07/01/16 Чтв 21:18:50 #57 №245717 
>>245699
Нет видео записи ни взлёта, ни посадки, ни возвращения. не были. Ноудискасс
Аноним 07/01/16 Чтв 21:24:05 #58 №245718 
>>245717
>Нет видео записи ни взлёта, ни посадки, ни возвращения
>ни взлёта
https://www.youtube.com/watch?v=L8FG0AMNmb4
>ни посадки
https://www.youtube.com/watch?v=Obd_jTO66-0
>ни возвращения
https://www.youtube.com/watch?v=b6TYTG81NtY
Вот примерно так кормят хуйцами поехавших.
Ты тралировал что ли?
Аноним 07/01/16 Чтв 23:45:08 #59 №245747 
14521995080350.jpg
Можем ли мы увидеть вселенную в момент большого взрыва или непосредственно после него?
Аноним 08/01/16 Птн 00:01:21 #60 №245750 
Правда ли что Вега - лучшая из легких ракет-носителей?
Аноним 08/01/16 Птн 00:04:39 #61 №245751 
>>245747
Если под "увидеть" понимается ЭМ-излучение, то скорее всего нет. Вселенная стала для него прозрачной в районе 350 тысяч лет после БВ, см. фоновое микроволновое излучение.
Аноним 08/01/16 Птн 01:38:41 #62 №245778 
>>245750
Что такое "лучшая"?
Аноним 08/01/16 Птн 01:40:09 #63 №245780 
>>245778
Дешевая
Аноним 08/01/16 Птн 01:45:43 #64 №245782 
Я не понимю, почему невозможна одновременно геостационарная и гелиостационарная орбита? То есть... короче, представьте циферблат для ориентации в пространстве. В центре находится Земля. В день, скажем, летнего солнцестояния, Солнце находится в направлении 12 часов. Запускаем спутник на орбиту Земли так, чтобы спутник находился тоже на 12 часах. Проходит время, Земля вращается, Солнце оказывается, скажем, на 5 часах, а спутник всё ещё на 12, но не потому что он уже не раз сделал обороты по циферблату и снова вернулся на 12, а потому что он там, ан 12 и стоял.
Так почему не может быть такого, чтобы спутник находился в гелиостационарной точке на геостационарной орбите?
Аноним 08/01/16 Птн 01:55:37 #65 №245784 
>>245782
Оборот вокруг своей оси Земля совершает за 24 часа, а вкруг Солнца за 365 дней, хули тебе не понятно?
Аноним 08/01/16 Птн 01:58:42 #66 №245785 
14522075224300.png
>>245522
Потому что железо внутри, железо снаружи!
Аноним 08/01/16 Птн 02:02:42 #67 №245788 
14522077627930.png
>>245784
Мне непонятно, почему спутник не может оставаться всегда в одном положении относительно Земли.
Аноним 08/01/16 Птн 02:13:41 #68 №245791 
>>245788
А разве это гелиостационарное положение в привычном понимании?
Аноним 08/01/16 Птн 03:05:21 #69 №245802 
>>245788
Потому что ему нужно двигаться, что бы не пиздануться на Землю обратно.
Аноним 08/01/16 Птн 03:54:27 #70 №245805 
>>245782
>гелиостационарная орбита
Нет такой орбиты, блядь. Солнце - не твердое тело с постоянной угловой скоростью вращения по всей поверхности, а газовый шар, вращающийся хуй пойми как. Ты походу вообще не понимаешь сути стационарных орбит и тот факт, что спутник на ГСО движется по круговой орбите со скоростью обращения Земляшки вокруг своей оси, а не просто в пустоте висит.
Аноним 08/01/16 Птн 03:59:00 #71 №245806 
>>245802
То есть типо скорость неразрывно связана с тем, чтоб не упасть под воздействием гравитации?
Аноним 08/01/16 Птн 05:14:44 #72 №245810 
>>245806
Бля. Это конечно ликбез, но все же.
Аноним 08/01/16 Птн 05:17:08 #73 №245811 
>>245810
Я тупой если, блять. Спасибо за ответ. Хотя это огрызок, а не ответ. Но я понял, что свзяано. Я просто думал достаточно выйти на окоземную орбиту и будешь там висеть, и заодно вместе с землёй двигаться
Аноним 08/01/16 Птн 05:21:58 #74 №245812 
>>245811
Движение по орбите это падение. Спутник не падает потому что его скорость очень высока. Для наглядности раскрути грузик на веревочке, пока будешь крутить у тебя будет орбита, замедлишь вращение - и твой спутник упадёт. Тут конечно разные причины но для наглядности сойдёт
Аноним 08/01/16 Птн 05:24:12 #75 №245813 
>>245812
Ладно, а возможно ли, тчобы спутник находился одновременно на гелиостационаре как >>245788
но при этом, формально был именно при Земле а не просто летел по орбите Земли рядом с Землёй?
Типа, если он будет очень высоко, на границе области гравитационного влияния планеты?
Аноним 08/01/16 Птн 05:28:40 #76 №245816 
>>245813
Я не совсем понял, возможно ты про это
https://ru.m.wikipedia.org/wiki/Точки_Лагранжа
Лететь по орбите земли он все равно будет
Аноним 08/01/16 Птн 05:35:26 #77 №245817 
>>245816
Нет, не про них. Просто думаю: если всё же добиться вращения спутника вокруг Земли именно по типу рисунка >>245788, то за счёт уменьшения скорости вращения спутника относительно Земли. Но это чревато падением спутника на планету. А что если просто поднять орбиту спутника так высоко от планеты, что он почти выходит за пределы гравитационного влияния Земли?
Представь планету, состоящую целиком и полностью из жидкой штильной воды и только ядро у неё твёрдое. Обычный корабль, судно, не космичсекий, будет всё время дрейфовать по поверхности такой планеты. и вот в этом примере - ядро жидкой планеты - Земля, вода жидкой планеты - её гравитационное поле. А граница этого поля - как раз то место, куда поместить спутник, чтобы он "дрейфовал" на бистационарной орбите.
Аноним 08/01/16 Птн 05:41:32 #78 №245818 
>>245817
Нахуя это все.
Аноним 08/01/16 Птн 05:47:51 #79 №245819 
>>245818
Потому что душа ХОЧЕТ видеть спутники Земли на гелиостационарной и одновременной геостационраной орбитах. Понимаешь?
Аноним 08/01/16 Птн 05:57:39 #80 №245821 
>>245819
Нет
Аноним 08/01/16 Птн 07:58:43 #81 №245837 
Чего добился роскосмос за последние 25 лет? программы, открытия, хабл, уебб, что угодно.
Аноним 08/01/16 Птн 07:59:15 #82 №245838 
>>245837
Ничего. Только дети директоров стали жить лучше.
Аноним 08/01/16 Птн 08:03:31 #83 №245839 
>>245838
То есть они паразиты?
Аноним 08/01/16 Птн 08:05:02 #84 №245840 
>>245839
Именно!
Аноним 08/01/16 Птн 08:38:14 #85 №245843 
>>245819
>гелиостационарной и одновременной геостационраной орбитах
Во-превых, на картинке >>245788 не гелиостационарная орбита.
Как писали выше - стационарная орбита = висение над одной точкой поверхности. У Солнца с этим сложно, т.к. слои вращаются с разной скоростью, но возьмём 25 дней, как скорость вращения слоёв на экваторе1.
У нас взаимоисключающие параграфы: гелиостационар требует обращения вокруг Солнца за 25 дней, геостационар - вокруг Земли за 24 часа. Разные тела, разные скорости и зонд сгорит нахер, будучи гораздо ближе к Солнцу, чем Меркурий.
На твоей картинке >>245788 не гелиостационар, а хуй пойми что.
Для того, чтобы объект обращался вокруг Земли за год (т.е. находился на одном положении, как на картинке), надо, чтобы высота орбиты составляла 2 миллиона километров2, что дальше точки Лагранжа L1 или L2. Такой орбиты нет. То есть, можно её представить, если взять сферическую Землю в вакууме, одинокую на сотни световых лет во всех направлениях, но у нас и Солнце, сила притяжения которого варьируется от 11410-6м/с2 3 до 11810-6м/с2, делая орбиту невозможной, а на таких расстояниях можно вспомнить ещё и про влияние Юпитера.
Ну и сложно придумать цель для зонда в таком положении, кроме, разве что, "чаяния души" какого-нибудь поехавшего.
1http://nssdc.gsfc.nasa.gov/planetary/factsheet/sunfact.html
2http://hyperphysics.phy-astr.gsu.edu/hbase/orbv3.html
3https://ru.wikipedia.org/wiki/%D0%A2%D0%BE%D1%87%D0%BA%D0%B8_%D0%9B%D0%B0%D0%B3%D1%80%D0%B0%D0%BD%D0%B6%D0%B0#L1
Аноним 08/01/16 Птн 08:40:54 #86 №245844 
>>245843
>11410-6м/с23 до 11810-6м/с2
114х10-6м/с2 до 118х10-6м/с2
Квикфикс.
Грёбаная макака. Выпилить бы эти костыльные маркеры разметки и оставить только бб-код.
Аноним 08/01/16 Птн 10:59:49 #87 №245869 
>>245837
МКС, Радиоастрон. Всё.
Аноним 08/01/16 Птн 11:00:38 #88 №245870 
>>245869
МКС совместный проект. Давай только то, что в соло сделоли.
Аноним 08/01/16 Птн 11:01:57 #89 №245871 
>>245870
Тогда только Радиоастрон. Ну там еще всякие Спектры пилятся понемногу, но они пока не взлетели.
Аноним 08/01/16 Птн 11:04:19 #90 №245873 
>>245871
Вот он, УСПЕХ! Хотя у муррики, хз даже, что у неё там в соло есть. Кроме марсохода. А так то дохуя. Сколько всякой штуки улетело и фоточки присылает. А мы чего запустили за тот год? Ммм, нихуя.
Аноним 08/01/16 Птн 11:11:34 #91 №245875 
>>245873
> Хотя у муррики, хз даже, что у неё там в соло есть.
Спирит/Оппортьюнити/Куриосити, куча всякой хрени на орбите Марса, LRO, Чандра, Спитцер, Кеплер, Галилео, Новые Горизонты - это то, что сходу вспомнилось из того, что НАСА делало в основном соло (хотя, емнип, на Куриосити есть российский инструмент, например), а не в сотрудничестве, как Кассини-Гюйгенс, например.
Аноним 08/01/16 Птн 11:14:12 #92 №245876 
>>245875
Да я же вспомнил дальше, лол. Понятно, что на порядок больше проектов и всяких вундевафель. Плюс да, столько всяких штук летает по системе. Чому мы такие запускать не можем? Ведь не тупее амеров.
Аноним 08/01/16 Птн 11:50:16 #93 №245881 
>>245876
Потому что амеры получают прибыль от продаж и масс медиа, а мы нет. А знание что там у плутона за азот, жидкий или дождеобразный, для прикладной науки ненужно. Всё это продолжение той влаговтыкательной борьбы с СССР, которой уже нет в помине.
Это всё справедливо для всего кроме марса. На марсе да.
Аноним 08/01/16 Птн 11:58:23 #94 №245883 
>>245881
Выходит, наса окончательно скоммерциализировалось и уже возвращается?
Аноним 08/01/16 Птн 12:14:46 #95 №245884 
>>245875
Инструменты не считаются, на практически любой АМС летает международная нагрузка. Алсо, наса не дебилы и не делают все "самостоятельно", нет такого понятия. На любом аппарате в мелочах у них иностранных подрядчиков масса, в основном из европки (как и в радиоастроне, разрабы которого тоже вполне себе не долбоебы).
Аноним 08/01/16 Птн 12:21:03 #96 №245886 
>>245883
Не ради денег летали только в СССР, и то только на бумаге.
Аноним 08/01/16 Птн 12:22:14 #97 №245887 
>>245886
И это чистая правда. В ссср была ИДЕЯ. Муррика же изначально капиталистическая страна, и я не говорю, что это плохо или что совок плохо. Просто разные подходы.
Аноним 08/01/16 Птн 12:22:28 #98 №245888 
14522449482720.jpg
>>245881
> Новые Горизонты - продолжение флаговтыкательной борьбы с ССсссс
Блять, кто впустил на спейсач этого идиота?
Аноним 08/01/16 Птн 12:24:12 #99 №245889 
>>245718
Вот эта хуйня плохого качества и есть пруф. Бля...
Аноним 08/01/16 Птн 12:25:52 #100 №245890 
>>245887 >>245886 >>245883 >>245881
Как можно быть настолько тупыми? Уёбывайте в загон.

>>245718
>Вот примерно так кормят хуйцами поехавших.
Нет, хуйцами кормят посылая в зогач зеленое/залетное быдло.
Аноним 08/01/16 Птн 12:28:29 #101 №245891 
>>245890
Причём тут загон, даун. У двух стран был кардинально разный строй и подход к делу. Это, надеюсь, ты отрицать не будешь? И сам пиздуй в загон, долбоеб. Хоть бы дочитал до конца, чтоб свой высер тут постить.
Аноним 08/01/16 Птн 12:37:01 #102 №245893 
>>245891
Я дочитал все до конца и шлю тебя нахуй, а твоей мамаше-шлюхе я вчера на ебло насрал, пока ее ебал, и она еще просила, потому что твой папка так делал, мразь залетная.

И так засрали единственную нормальную доску сосаки своими заслугами-покаяниями-дидами-выяснениямиукоготолще, ублюдки лоботомированные блять, теперь и в тред тупых вопросов пришли.
Аноним 08/01/16 Птн 12:38:39 #103 №245894 
>>245893
Таблетки выпей, болезненный. Никто тут желчь кроме тебя не льёт, даун. Все вели нормальный диалог, пока ты со своей желчью сюда не припёрся. Никто тут про дедов ничего не писал, очнись, долбоёб.
Аноним 08/01/16 Птн 13:01:20 #104 №245900 
>>245889
Говорю же - поехавший.
Ты хотел Ultra HD качества от телекамер 70-х годов что ли, уёбок?
Аноним 08/01/16 Птн 13:02:23 #105 №245901 
>>245900
Да ясно, я поверил уже. Всё, были, летали, садились. Уймись уже.
sageАноним 08/01/16 Птн 14:00:28 #106 №245913 
>>245889
Посмотри все серии When We Left Earth. Там дохуя архивных плёночкек хд качества и божественный саундтрек.
Аноним 08/01/16 Птн 14:30:43 #107 №245919 
Возможно ли посчитать площадь круга.
Аноним 08/01/16 Птн 14:34:14 #108 №245920 
>>245919
Площадь - нельзя, только объём.
Аноним 08/01/16 Птн 14:34:56 #109 №245921 
>>245920
Объём круга? Это как?
Аноним 08/01/16 Птн 14:35:37 #110 №245922 
>>245921
Ноль. Он же плоский!
Аноним 08/01/16 Птн 14:36:56 #111 №245924 
>>245698
>Я не ошибся
А Гагарин был? Почему, где доказательства?
Аноним 08/01/16 Птн 14:37:55 #112 №245925 
>>245922
Ну и как ноль относится к объёбу?
Аноним 08/01/16 Птн 14:38:11 #113 №245926 
>>245925
*объёму
Аноним 08/01/16 Птн 14:40:40 #114 №245927 
>>245925
> Ну и как ноль относится к объёбу?
Прямо. Объебм - скалярная величина, значит имеет полное право быть нулевым.
Аноним 08/01/16 Птн 14:46:07 #115 №245929 
>>245869
>МКС
Её же вроде процентов на 80 американцы построили "Шатлами" всеразличными.
Аноним 08/01/16 Птн 14:49:08 #116 №245932 
>>245821
а он есть
Аноним 08/01/16 Птн 15:01:23 #117 №245935 
>>245927
мне кажется даже, что если бы он был веторной величиной в 12 мерном пространстве времени комплексных зеленых слонов, то при определенном на этом множестве кольце, он имел бы право быть нулевым.
Аноним 08/01/16 Птн 16:03:53 #118 №245953 
14522582337360.png
>>245254 (OP)
ай, мимо кассы. ладно, хорошо, что заметил. дубль два:
сап, анон. никак не могу сесть готовиться к очередному экзамену, поэтому возник такой тупой вопрос:
человечество уже не один десяток лет строит ракеты и пуляет их в космос. с полёта первого человека в космос прошло без малого почти 55 лет. за эти годы камплуктер из огромной ебанины с мощностью калькулятора превратился в скромную пекаренку а современные смартфоны по мощности уже равны компьютерам нулевых, на 3Д-принтерах уже потихоньку печатают человеческие ткани и жилые дома, автомобили через пару десятков лет станут беспилотными, через 10 лет начнутся эксперименты на ИТЭРе. так объясни мне, анон, если сможешь, почему ракетостроение до сих пор не поставлено на конвейер? почему ракеты не клепают машины, а до сих пор преимущественно собирают люди? в чём техническая сложность сделать автоматизированный цех по сборке РН и КА? очень надеюсь на более развёрнутый ответ, чем "нет задач" или "нахуй не нужно столько ракет, чтобы для них сборочную линию делать".
Спасибо.
Аноним 08/01/16 Птн 16:22:00 #119 №245966 
>>245953
> через 10 лет начнутся эксперименты на ИТЭРе
а на виндельштейне уже начались, немцы дали пасасать всему миру
> ракетостроение до сих пор не поставлено
Ты просто не знаешь что такое конвейер. Конверйер - это место где работают люди.
А то про что ты хочешь спросить - это ГАП, гибкое автоматизированное производство.
Но его тонет не только для ракет, но вообще ни для чего. Есть отдельные ГПЛ, гибкие производственные линии. Для микропроцессоров, для патронов. Это более востребованные вещи. Но надо пони мать, что и для них и для чего угодно сначала возник спрос, а потом только ГПЛ. В общем степень автоматизации производства диктуется необходимой массовостью продукта, но не наоборот.
Поэтому как бы ты ни хотел поучить какой-то другой ответ на свой ответ, ты получишь только "нет задач".

Другими словами - хочешь безлюдное производство ракет - сделай его сам.
Аноним 08/01/16 Птн 16:35:34 #120 №245968 
>>245953
Фактический, нужно сначала построить завод по производству неведомой ебаной херни а орбите, причем такой, которая была бы нужна ВСЕМ как воздух или даже нужнее, а только потом построятся конвейеры, чтобы подтаскивать туда ракетами заготовки и спускать продукцию.

Сейчас на орбите работает лишь маленькая лаборатория и мастерская. При этом науку скидывают обратно по ваюфаю, а продукцию мануфактуры - грузовиком.
Аноним 08/01/16 Птн 16:38:35 #121 №245969 
>>245953
>нахуй не нужно столько ракет, чтобы для них сборочную линию делать
Но ведь именно таков верный ответ.
Чем ты собираешься захламлять орбиту, чтобы тебе понадобилось больше ракет, чем сейчас, и чем ты платить собираешься?
Если вдруг объявится миллиардер космофил и заплатит свое состояние за постройку дворца на орбите - ему запилят. Не сразу, надо будет выйти на большие производственные мощности, чем сейчас, может даже сверхтяж высрут, но сделают, если заплатит.
А пока полтора зонда в ебеня, десяток-другой коммуникационных/военных/метео спутников, да полдюжины рейсов до МКС. При такой нагрузке строят столько РН, сколько нужно. Если наращивать объемы производства РН, то куда их девать? Хранить в ангаре, чтоб крышей приложило? Продавать со скидкой корейцам?
Аноним 08/01/16 Птн 16:49:35 #122 №245974 
>>245966
>>245968
>>245969
спасибо, анон, за развёрнутые ответы. я понимаю, что краткость - сестра таланта, но я хотел услышать развёрнутые мнения по этому вопросу. я их получил и доволен. сам вопрос возник просто к тому, что у одной России запусков ~20-25 в год, и для ГПЛ, по-моему, этого достаточно. всё-таки не айфоны будут делать, а РН и КА, а автоматика повысит уровень надёжность и уменьшит количество отказов в моём понимании.
Аноним 08/01/16 Птн 16:56:23 #123 №245979 
Где найти подробную инфу по профилю полета, двигателям, запасу топлива ракеты Р-29РМ? Или это закрытая инфа?
Аноним 08/01/16 Птн 17:08:10 #124 №245983 
>>245974
> и для ГПЛ, по-моему, этого достаточно.
Нет, недостаточно. ГПЛ будет производить одну ракету в час, например. Остальное время - простаивать. Это невыгодно с точки зрения чего угодно.
И вообще, ур500 собирали именно на конвейере. Люди собирали, но элементы конвейера присутствовали. Был специальный человек для закручивания одной гайки специальным ключом. Он закручивал сто гаек в день, например. Конвейер - он вот какой-то такой.

Почему это стало возможным? Просто НОО на тот момент оказалась самым высоким, удобным и чуть ли не единственным подходящим деревом, с которого голые обезьяны могли бы сбрасывать самые большие кокосы на головы других голых обезьян, которые отказывались жить так же как первые, и говорить на их языке. Эта психология голых обезьян может объяснить очень многое.
А вот луна оказалась слишком высоким деревом, и не слишком удобным.
Аноним 08/01/16 Птн 17:12:32 #125 №245987 
Почему никто ещё не сделал ракету на пентаборане + трифлориде хлора с импульсом за 400?
Аноним 08/01/16 Птн 17:14:01 #126 №245988 
>>245987
Гроб-гроб-фтор-фтор-пентаборан-кладбище?
sageАноним 08/01/16 Птн 17:14:42 #127 №245989 
>>245988
>фтор-хлор
быстрофикс
Аноним 08/01/16 Птн 17:22:22 #128 №245993 
>>245987
Такую-то крылатую ракету япошки могли бы запилить, но только в условиях войны.
А в мирное время - нинужна. В среднем если посмотреть историю, человеческая жизнь дорожает. Ну и здоровье - тоже.
Аноним 08/01/16 Птн 17:28:22 #129 №245998 
14522633027970.png
>>245953
Но союзы и так клепают за 9 месяцев от поставки материалов до ракеты на старте. Это и есть ёбаный конвеер, учитывая какую огромную работу надо проделать. И с мобилками это в принципе не сравнить, потому что совершенно разные требования ЕСКД. Мобилки только при комнатных условия должны работать, а тут каждую деталь нужно подвергнуть полной диагностике, иначе из-за малейшего дефекта вся ракета идет по пизде. Это стоит огромных денег. В авиации тоже самое с полыми винтами в турбинах. Для них такие технологии требуются, плюс нейтронография в ёбаном ядерном реакторе, что дешевле из чистого золота отлить.
Аноним 08/01/16 Птн 17:47:25 #130 №246000 
>>245254 (OP)
Почему ракеты не делают "каплевидной" вормы? Это ведь форма с наименьшим сопротивлением.
Аноним 08/01/16 Птн 17:54:13 #131 №246005 
>>246000
Рисуй "каплевидную" ворму, обоссым.
Аноним 08/01/16 Птн 17:57:06 #132 №246007 
14522650266150.png
>>246000
1. Это форма с наименьшим сопротивлением только на дозвуке. На сверх- и гиперзвуке оптимальная форма совсем другая, и большинство ракет ей приблизительно соответствует.
2. Атмосферные потери достаточно малы, чтобы не выдрачивать форму до идеальной.
Аноним 08/01/16 Птн 18:00:29 #133 №246009 
>>246005
Яйцеобразная тогда.

>>246007
На сверх и гипере оптимальная как на картиночке?
Аноним 08/01/16 Птн 18:36:52 #134 №246031 
>>246009
На сверхзвуке да. https://en.wikipedia.org/wiki/Sears–Haack_body

Гиперзвук - он бывает сильно разный, в зависимости от плотности и скорости есть несколько термодинамических режимов, в которых сильно меняется состав и поведение среды. Но ракеты обычно выскакивают из атмосферы на "низком" гиперзвуке, в котором тело Сирса-Хаака тоже оптимально.
Аноним 08/01/16 Птн 20:53:57 #135 №246070 
А чем отличается Удельный импульс от тяги? Было бы шикарно, если попонятнее о каждом. Можно математическим языком.

И другой вопрос немного не в тему . ВКО В ВКС чтоле переименовали?
Аноним 08/01/16 Птн 20:56:28 #136 №246071 
>>246007
> 1. Это форма с наименьшим сопротивлением только на дозвуке. На сверх- и гиперзвуке оптимальная форма совсем другая, и большинство ракет ей приблизительно соответствует.
А еще у ракеты есть двигатели, которые выбрасывают очень много газа в корме, понижая тем самым сопротивление.
А еще технологичность тоже не на последнем месте.
Аноним 08/01/16 Птн 21:11:22 #137 №246075 
На прошлый мой тупой вопрос никто не ответил, скопипащу его сюда.

Есть у меня друг старый, который как-то задал мне туповатый вопрос, на который я не смог ответить. Информацию по этому вопросу найти тоже не удалось. Тащемта материя имеет свой конкретный цвет. Молекула таки имеет цвет (вроде должна), ибо является наименьшей составляющей материи, которая сохраняет её свойства. Внимание вопрос: какого бля цвета атомы, и есть ли у них цвет вообще? Если они все же имеют определенный цвет, то какого цвета могут быть еще более мелкие частицы?
Аноним 08/01/16 Птн 21:28:54 #138 №246080 
>>246070
Есть много разных единиц измерений для УИ, но простое и понятное определение можно записать так:

Удельный импульс = тяга / расход топлива в единицу времени.

Часто приводят значения УИ в секундах. УИ в секундах = предыдущее определение / гравитационное ускорение на поверхности земли.

http://www.wolframalpha.com/input/?i=3500+newtons+%2F+%281+kg+%2F+s%29+%2F+earth+gravity

^ формула, описывающая УИ двигателя с расходом топлива 1 кг/с и тягой 3500 Н
Аноним 08/01/16 Птн 21:32:17 #139 №246083 
>>246075
>Молекула таки имеет цвет
Нет. В той формулировке, в которой ты понимаешь "цвет" у молекулы его нет.
Простой пример тебе, чтобы понять сразу: углерод в виде графита чёрный а в виде алмаза бесцветный и прозрачный.
Аноним 08/01/16 Птн 21:45:55 #140 №246087 
>>246075
>какого бля цвета атомы, и есть ли у них цвет вообще?
Черный, что эквивалентно "не имеет цвета". Длина волны света меньше, чем размер атома, вот и нет у них цвета.
Аноним 08/01/16 Птн 21:55:32 #141 №246093 
>>246080
слыш мамкин физик, УИ - это прежде всего скорость истечения реактивной струи газов из сопла. Скажем у водородного движка так называемые 453с в вакууме, что соответствует скорости ~4490 м/с. Про формулы Циолковского не забываем. А расход горючки ты это уже частный случай берешь.
Аноним 08/01/16 Птн 21:58:27 #142 №246097 
>>246075
Цвет - это субъективная хуйня, существующая только у тебя в голове. Он может быть разным даже если ты грибов поешь. Так что будем считать что ты имеешь в виду спектральные характеристики в видимом диапазоне.

К сказанному анонами выше добавлю, что спектр может быть разным не только у веществ с разной молекулярной структурой (графит vs алмаз), но и с разной формой поверхности или макроструктурой (метаматериалы).
Аноним 08/01/16 Птн 22:06:33 #143 №246100 
>>246070
>удельный импульс
>попонятней
сколько секунд проработает движок на одном килограмме топлива, развивая тягу в один ньютон

чем больше - тем экономичней

> как связан с тягой
чем выше импульс, тем больше надо энергии, чтобы получить ту же тягу, практически говоря - чем движок экономичней, тем хуевей он тянет
Аноним 08/01/16 Птн 22:11:52 #144 №246101 
>>245987
Может кто и пытался сделать, только больше о нем никто не слышал.
Аноним 08/01/16 Птн 22:15:17 #145 №246103 
14522805170480.jpg
>>245953
Зачем астронавты выдавили лобовые стекла шаттла? Проветриться решили?
Аноним 08/01/16 Птн 22:17:34 #146 №246104 
>>246103
Фотощоп
Аноним 08/01/16 Птн 22:27:51 #147 №246107 
>>246104
Даже на сайте NASA?
http://spaceflight.nasa.gov/gallery/images/shuttle/sts-114/html/iss011e11227.html
Аноним 08/01/16 Птн 22:29:17 #148 №246108 
>>246107
Да
Аноним 08/01/16 Птн 22:32:15 #149 №246111 
>>246108
Вот суки, небось Кубрик помогал.
Аноним 08/01/16 Птн 22:43:40 #150 №246113 
>>246103
Там все нормально
Аноним 08/01/16 Птн 23:28:34 #151 №246137 
>>245987
>на пентаборане + трифлориде хлора
Где там гептилошизик, он бы оценил идею.
Аноним 08/01/16 Птн 23:52:35 #152 №246141 
>>246080
>>246100
>>246093
Cпасибо.
Аноним 09/01/16 Суб 00:08:04 #153 №246146 
14522872842430.png
А в чём собственно бургут от гептила? Вроде ну отрастят 3 лёгкое Казахстанцы, но ведь они и не помешают сделать уже из пентаборана ракету. Или я переоцениваю уровень жлобства в мире?
Аноним 09/01/16 Суб 00:45:45 #154 №246150 
>>246146
Опасность гептила слишком преувеличена
Аноним 09/01/16 Суб 00:52:33 #155 №246152 
>>246137
А в чем профит такого топлива?
Аноним 09/01/16 Суб 01:11:55 #156 №246155 
>>246152
Профит в высоком удельном импульсе. Ну, если как-нибудь умудрится такую ракету заправить и запустить двигатель без того, чтобы она взорвалась.
Аноним 09/01/16 Суб 01:29:40 #157 №246168 
>>246155
Я думаю, главное технику безопасности не нарушать. А какой импульс?
Аноним 09/01/16 Суб 01:41:14 #158 №246171 
>>246168
Ну, с тетроксидом азота 365 секунд, значит с трифлоридом будет ещё выше, под 380.
Аноним 09/01/16 Суб 01:52:04 #159 №246172 
>>246171
Так на водород-кислород вроде так же.
Аноним 09/01/16 Суб 02:53:56 #160 №246187 
Большое спасибо за пояснения.
Аноним 09/01/16 Суб 03:40:50 #161 №246192 
>>246172
какая только плотность у жидкого водорода посмотри
Аноним 09/01/16 Суб 10:14:23 #162 №246211 
>>246171
> 380
Метан-кислород почти столько дает, не?
Алсо, нашел патент с этиленом вместо большей части метана, почему на практике никто такое не рассматривает?
http://ru-patent.info/21/80-84/2180050.html
Аноним 09/01/16 Суб 12:50:25 #163 №246226 
>>246100
хуево все сказал

чем больше импульс тем меньше реактивного тела требуется выкинуть и тем больше энергии надо потратить на создание тяги в 1 Ньютон
Аноним 09/01/16 Суб 13:17:23 #164 №246235 
>>245966
>В общем степень автоматизации производства диктуется необходимой массовостью продукта, но не наоборот.
это не так, на данный момент могут себе позволить, а не массовость диктует.

>Другими словами - хочешь безлюдное производство ракет - сделай его сам.
а вот это тру.
Аноним 09/01/16 Суб 14:09:03 #165 №246258 
>>246103
Вышли бы в космос, пробзделись. А то сидите как сычи в своих шатлах.
Аноним 09/01/16 Суб 14:12:26 #166 №246261 
>>245254 (OP)
Какое самое самое эффективное твердое топливо для ракет?
Какое самое безопасное твердое топливо?
Какое самое оптимальное по своим характеристикам, стабильность, теплота сгорания, тяга, т.д. твердое топливо?
Аноним 09/01/16 Суб 14:55:13 #167 №246283 
>>246258
Влом как то.
Аноним 09/01/16 Суб 15:10:50 #168 №246290 
>>246087
Больше.
Аноним 09/01/16 Суб 15:36:08 #169 №246299 
14523429682270.jpg
Нахуя наса пишет на баллоне, что кислород не горюч? Да, ФОРМАЛЬНО сам жидкий кислород не горит, но он ведь дохуя пожаро- и взрывоопасен, не?
Аноним 09/01/16 Суб 15:39:57 #170 №246301 
>>246146
Не понял картинки. ЧТо плохого в названии Мир?
Аноним 09/01/16 Суб 15:46:35 #171 №246305 
>>246299
Способствует пожару, но не его причина. Горит керосин, кислород икисляет.
Ну и конечно
https://ru.wikipedia.org/wiki/%D0%92%D0%B7%D1%80%D1%8B%D0%B2_%D1%80%D0%B0%D1%81%D1%88%D0%B8%D1%80%D1%8F%D1%8E%D1%89%D0%B8%D1%85%D1%81%D1%8F_%D0%BF%D0%B0%D1%80%D0%BE%D0%B2_%D0%B2%D1%81%D0%BA%D0%B8%D0%BF%D0%B0%D1%8E%D1%89%D0%B5%D0%B9_%D0%B6%D0%B8%D0%B4%D0%BA%D0%BE%D1%81%D1%82%D0%B8
Аноним 09/01/16 Суб 15:51:34 #172 №246308 
>>246301
На пикче запланированный свод с орбиты после окончания эксплуатации. Картинку делала полная бестолочь.
Аноним 09/01/16 Суб 16:15:23 #173 №246310 
>>246308
Ты не ответил на вопрос.
>Что плохого в названии "Мир"?
Аноним 09/01/16 Суб 16:29:47 #174 №246316 
>>246310
Это такая шутка.
Аноним 09/01/16 Суб 16:52:35 #175 №246325 
14523475559300.jpg
>>246316
Аноним 09/01/16 Суб 17:17:37 #176 №246328 
Будет ли эффективен телескоп, основанный на линзировании ЧД?
Аноним 09/01/16 Суб 17:24:52 #177 №246329 
>>246328
Поясни что это
Аноним 09/01/16 Суб 17:44:14 #178 №246335 
>>246329
Ну телескоп в котором линзируется через ЧД.
Аноним 09/01/16 Суб 17:59:44 #179 №246349 
>>246328
> Будет ли эффективен телескоп, основанный на линзировании ЧД?
Будет. Более того - астрономы вовсю пользуются известными случаями гравитационного линзирования. Другое дело что поуправлять этим, естественно, не выходит, и приходится пользоваться тем, что нашли.
Аноним 09/01/16 Суб 19:05:11 #180 №246387 
И как далеко ЧД увеличивает? Скажем если Планета N находится на расстоянии 10 пк. от ЧД мы от неё на достаточном для рассмотрения расстоянии. Что бы мы увидели?
Аноним 09/01/16 Суб 19:09:22 #181 №246389 
>>246387
Насколько я знаю, "гравитационная линза" это просто словесная абстракция. Она ничего не увеличивает, она просто изгибает мимокрокодильный свет от источника света за ней так, чтобы он вместо ебеней попал на земляшку.
Аноним 09/01/16 Суб 19:43:42 #182 №246393 
Два вопроса:
1. Был ли бы более успешен шаттл грузоподъёмностью 5-6 тонн? То есть я так понимаю он был бы мень, дешевле, проще в обслуживании, и вместо одного большого можно было бы запилить два-три маленьких и возить мелкоспутники
2. Почему космическая программа СССР была сконцентрирована на Венеру?
Аноним 09/01/16 Суб 19:52:34 #183 №246394 
>>246393
1. Логика подсказывает, что целесообразнее иметь многоразовую BOLSHO YOBA, чем многоразовую MALIY YOBA. А для мелкоспутников есть одноразовые карандаши, производство которых давно отточено.
Аноним 09/01/16 Суб 20:18:37 #184 №246405 
>>246393
он заменял им орбитальную станцию

интересный вопрос, потому что ближе ?
Аноним 09/01/16 Суб 20:43:48 #185 №246415 
>>246405
Ну нет. Шаттл изначально пилился ДЛЯ снабжения и сборки новой орбитальной станции. Потом оказалось что денюжек и на и на то нету, станцию похерили, шаттл оставили.
Имхо, самая большая ошибка была в отсутствии беспилотного режима (привет Буран). Так, изначально межполетное обслуживание занимало полтора месяца, после гибели челленджера, ковырялись уже три. Опять же, имхо, надо было иметь BOLSHAYA BESPILOTNAYA YOBA для заброски тяжёлых грузов и MALAYA YOBA для снабжения станций, смены экипажей и ремонта разных хаблов.
Аноним 09/01/16 Суб 20:45:40 #186 №246417 
>>246393
>2. Почему космическая программа СССР была сконцентрирована на Венеру?

С чего ты взял что космическая программа СССР была сконцентрирована на Венеру? Космическая программа СССР и на Марс была сконцентрирована.

Просто например из 4-х аппратов запущенных к Марсу в 1973 году ни один миссию не выполнил (Марс - 5,6,7,8) потому что зафейлили. Полимеры в нашей стране были прополимерены задолго до твоего рождения, анон.
Аноним 09/01/16 Суб 20:47:54 #187 №246419 
>>246415
>Имхо, самая большая ошибка была в отсутствии беспилотного режима (привет Буран).

Опять эта некробуранофилия. Вся автоматика на борту Шаттла стояла, включая систему автоматической посадки.

Это не ошибка - это исключительно политическое давление.
Аноним 09/01/16 Суб 21:04:51 #188 №246424 
>>246393
>1. Был ли бы более успешен шаттл грузоподъёмностью 5-6 тонн? То есть я так понимаю он был бы мень, дешевле, проще в обслуживании, и вместо одного большого можно было бы запилить два-три маленьких и возить мелкоспутники

Тут два анона ничего не понимающие в шаттлах написали что нет, но на самом деле вполне можно так предполагать. Но не из за мелкоспутников. И да, не факт что так бы случилось.

Размеры шаттла раздувались в ходе перепиливания ТЗ и постоянных споров, что в свою очередь приводило к еще большему раздуванию ТЗ и большим спорам. ТЗ на шаттл это типичный такой пример кластер фаkапа - почитай, рекомендую, очень интересно. Например очередное увеличение веса и габаритов произошло когда выяснилось что нужна поддержка пентагона. Пентагон согласился отдать свои деньги но потребовал что бы можно было запускать тяжеленные спутники кейхоле, cross-range capability (возможность садится после одного витка при полярном старте). В результате к шаттлу присобачили (ненужные никому) крылья и раздули размеры что бы эти кейхолы туда влезали. Это привело к тому что надо было разгонять движки ну и так далее. И вот такая хуйня там творилась с самого начала проекта. Сам понимаешь, одно дело пилить 10тонник который сейчас есть у целой кучи стран и технологически гораздо проще, другое дело иобу которая на орбиту выводит ~90 тонн(вес орбитера с грузом)

При чем в результате пентагон от шаттла отказался и шаттл остался без шансов стать рентабельным в принципе- он был рассчитан что все пуски США пойдут через него.
Аноним 09/01/16 Суб 21:16:09 #189 №246428 
>>246424
Как это пентагон отказался, что, не было контрактов даже? Делали им уступки просто так, за "может быть"?
Аноним 09/01/16 Суб 21:17:00 #190 №246429 
>>246424
>cross-range capability
Не, это было впилено не по требованию пентагона:

>Landing cross range was a driver for abort once around and not the single orbit mission.

http://www.collectspace.com/ubb/Forum30/HTML/001340.html
http://forum.nasaspaceflight.com/index.php?topic=4998.0

Размер грузового отсека в НАСА тоже хотели побольше. http://history.nasa.gov/SP-4221/ch5.htm

В целом роль военных в дизайне шаттла несколько преувеличена.
Аноним 09/01/16 Суб 21:22:17 #191 №246431 
>>246428
>Как это пентагон отказался, что, не было контрактов даже? Делали им уступки просто так, за "может быть"?

Не за может быть - они в конгрессе поддержали шаттл. Иначе бы проект не прошел и не было бы шаттла вообще.
Аноним 09/01/16 Суб 21:47:00 #192 №246439 
>>246419
Поясни ту хуйню что ты написал
Аноним 09/01/16 Суб 21:51:19 #193 №246442 
>>246393
2. Не была, аппаратов на Луну и Марс не меньше было послано.
Аноним 09/01/16 Суб 22:45:15 #194 №246470 
А почему бы не смешать вместе горючее и окислитель прямо в баках? Получилось бы упрощение конструкции, как с монопропеллантом но с высоким УИ.
Аноним 09/01/16 Суб 22:51:46 #195 №246476 
>>246470
Смешай водород и кислород, хули.
Аноним 09/01/16 Суб 22:51:55 #196 №246477 
>>246470
Потому что твердотопливные петарды именно так и делают.
Потому что при смешивании гептила и амила они ВОСПЛАМЕНЯЮТСЯ
Потому что температура хранения жидкого водорода и кислорода разная.
И т.д. и т.п.


Аноним 09/01/16 Суб 23:08:55 #197 №246502 
>>246477
Озон + метан оба жидкие при -170
Аноним 09/01/16 Суб 23:29:49 #198 №246527 
>>246415
по факту, а не в том что они там хотели или нет
ну и в конце концов прикотиллоосссь.
Аноним 09/01/16 Суб 23:55:35 #199 №246541 
>>246502
Но если их смешать они ёбнут, разве нет?
Аноним 09/01/16 Суб 23:57:40 #200 №246543 
14523730607220.png
А вот объясни мне умны анон. Звезда О класса это молодая звезда. Черная дыра это старая и уже не звезда. Как они могут быть одного возраста?
Сорви покровы с Брабена а то че он внатуре.
Аноним 09/01/16 Суб 23:58:33 #201 №246545 
14523731133760.png
>>246543
Вторая пикча отклеилась.
Аноним 10/01/16 Вск 00:00:23 #202 №246546 
14523732239190.png
>>246545
Епт. Перепутал. Вот правильно.
Аноним 10/01/16 Вск 00:06:20 #203 №246548 
Поясните за панспермию. Как бактерии без вреда для себя вылетают из гравитационного колодца?
Аноним 10/01/16 Вск 00:16:36 #204 №246551 
>>246261
Бемп вопросу.
Аноним 10/01/16 Вск 00:19:56 #205 №246552 
>>246470
Ты только что твердотопливное горючее.
Аноним 10/01/16 Вск 00:21:43 #206 №246553 
>>246543
Что такое возраст черной дыры?
Аноним 10/01/16 Вск 00:22:36 #207 №246554 
>>246546
>>246545
>>246543
Ошибка в коде, описание даунское или еще что. Мол, там сидит коричневый карлик, который еще со времен царя гороха тухнет, потом на него внезапно облака водорода, тысячи их и пошло поехало.
Аноним 10/01/16 Вск 00:23:36 #208 №246555 
>>246548
Так же как и излучение хокинга.
Почитай внимательнее про эту теорию. Никто протожизнь в колодцы не совает.
Аноним 10/01/16 Вск 00:32:33 #209 №246558 
>>246554
Но у Барабуха так устроена туманность NGC7822
Куча О звезд и многие в паре с черными дырами. Такое в принципе могло случится в эмиссионной туманности? Типа ебнула сверхновая и коричневые карланы возгорелись.
Аноним 10/01/16 Вск 01:01:30 #210 №246569 
14523768907580.jpg
>>246553
Это её возраст.
Аноним 10/01/16 Вск 01:12:17 #211 №246573 
Что насчёт Л. Ксанфомалити?
Похоже на пиздёшь. Похоже, потому что кроме 1 мутного пятна я не нашёл. А вы как думаете.
Аноним 10/01/16 Вск 01:41:45 #212 №246576 
14523793056840.jpg
>>246573
Я считаю что его ищет Оккам
Аноним 10/01/16 Вск 03:34:01 #213 №246598 
14523860420240.jpg
>>246389
Ну изгибание света и есть то, что делает обычная оптическая линза.
>>246387
>Планета N находится на расстоянии 10 пк
Что подразумевает эта планета? Мы на ней наблюдатели и она в десяти парсеках от ЧД, или мы наблюдаем линзирующуюся чд планету с Земли? Чтобы линза что-то увеличивала и мы могли это увеличение разобрать, наблюдение должно происходить из точки фокусного расстояния. Гипотетически, если бы все условия положения идеально совпали, то гравитационная линза могла бы давать многократное увеличение отдельных фрагментов отстоящего позади линзируемого объекта. Наблюдать же с Земли планету, отстоящую от линзирующей чд на 10 парсек позади, если сама система "линза-линзируемый объект" находится достаточно далеко(10 000 св. лет скажем), вовсе не удастся. Планета практически не излучает свет, чтобы мы могли ее наблюдать, а фокусное расстояние будет слишком удалено, чтобы давать увеличение более нескольких процентов. Увеличение зависит от фокусного расстояние, чем оно меньше, тем больше увеличение удаленного на бесконечность объекта.
А вообще гравилинзы - не оптические линзы, поэтому их увеличение работает совсем не так, как увеличение лупы. Это в любом случае будет искаженное, хоть и увеличенное изображение. Вообще я может полную хуету написал, но вроде нет.
Аноним 10/01/16 Вск 04:23:58 #214 №246600 
анон, а как бы это повлияло на солнечную систему, землю и лысых обезьян на ней, если бы нашу солнечную систему выкинуло в межгалактическое пространство?
Аноним 10/01/16 Вск 04:54:10 #215 №246601 
>>245254 (OP)
хэллоу
Аноним 10/01/16 Вск 05:12:33 #216 №246604 
>>246600
Ну скорее, кроме изменения картины звездного неба, особой разницы заметно бы не было. Вопрос скорее в зависимости нынешних орбит от гравитации в текущем секторе галактики. Сильно ли эта гравитация влияет на стабильность орбит? Но, вроде не сильно, так что даже орбиты особо не шатнутся.
Аноним 10/01/16 Вск 06:02:34 #217 №246607 
Тупой вопрос: Почему в шапку перестали пихать webm-ки?
Аноним 10/01/16 Вск 06:05:37 #218 №246608 
>>246607
У них значительно большая масса, чем у картинок, которая вызывает смещения орбит различных тредов и разделов, что увеличивает вероятность столкновения этого треда с ними. Такой ответ тебя устроит?
Аноним 10/01/16 Вск 06:07:16 #219 №246609 
>>246604
Есть ещё предположение, что местный пузырь защищает нас ака озоновый слой. И жизнь была бы не возможна если бы мы вместе по орбите бы не двигались.
Аноним 10/01/16 Вск 06:13:00 #220 №246611 
Как спускаемые аппараты на такие планеты как Европпа или ганимед планируют бурить многокилометровый лёд с неизвестными примесями?
Аноним 10/01/16 Вск 06:16:23 #221 №246612 
>>246611
Я понимаю, что это плохой ответ, но пока никак. Одна из причин, по которой эти аппараты еще даже не проектируются.
Аноним 10/01/16 Вск 06:19:06 #222 №246613 
>>246611
http://geektimes.ru/post/231399/
Аноним 10/01/16 Вск 07:04:03 #223 №246615 
>>246613
Как же они там сигнал передавать собрались? Даже тонюсенькая нить, которая обязательно порвётся, будет весить как 10 таких аппаратов. А если использовать метод всплытия то так же не получится ибо замёрзнет.
Аноним 10/01/16 Вск 13:14:02 #224 №246662 
>>246558
Вон смотри, два мужика в инвалидных каталках, молодой и старый. Оба на колясках, но разница в возрасте - 40 лет.

Похуй на возраст. И вообще, пиздуй учить
https://ru.wikipedia.org/wiki/Звёздная_эволюция
Аноним 10/01/16 Вск 14:25:45 #225 №246723 
>>246261
Ну ответьте, че вы?
Аноним 10/01/16 Вск 14:26:51 #226 №246725 
>>246723
Да на эти вопросы тут годами отвечают, бля.
Аноним 10/01/16 Вск 14:34:01 #227 №246737 
>>246429

> Не, это было впилено не по требованию пентагона:

Щас. Сам то свой же линк читал?

> http://history.nasa.gov/SP-4221/ch5.htm

A once-around abort on a due-east launch from Cape Canaveral would not be too difficult; the craft might land at any of a number of sites within the United States. In the words of NASA's Leroy Day, "If you were making a polar-type launch out of Vandenberg, and you had Max's straight-wing vehicle, there was no place you could go. You'd be in the water when you came back. You've got to go crossrange quite a few hundred miles in order to make land." 34

The Air Force had ample opportunity to emphasize its desire for crossrange by working within the Joint Study Group that Paine and Seamans had set up to seek a mutually-acceptable shuttle design.

Такой большой cross range для single orbit нужен был только для военных полярных орбит, у наса с мысом канаверал все было в порядке.

Другие ссылки у тебя на форумы где один конкретный военный приводит не все факты и из этого делает неверный вывод спасая честь мундира.
Аноним 10/01/16 Вск 14:37:26 #228 №246746 
>>246723
Ты ебанулся?
Аноним 10/01/16 Вск 14:40:00 #229 №246750 
>>246261
которое у тебя есть
которого у тебя нет
которого у тебя нет и не будет

не ну вроде выше перетирали же топляки не ?
но вообще на вики и смотри бустеры шаттла - и не иби мозг.
Аноним 10/01/16 Вск 14:44:19 #230 №246753 
>>246615
не будет весить как 10 аппаратов, сделают из карбона
вообще гугли управляемые торпеды
Аноним 10/01/16 Вск 15:28:21 #231 №246799 
>>246439
>Поясни ту хуйню что ты написал

На шаттле были автоматизированы все процессы (по отдельности) включая посадку. Так что приводить тут Буран в пример это просто разновидность дрочки.

Однако

а) испытать полностью автоматическую посадку не дали пилоты - беря управление на себя (саботировав?).

б) заказа на полностью автоматические миссии не было от слова совсем. То есть все равно летали экипажами бы.
Аноним 10/01/16 Вск 15:50:50 #232 №246809 
14524302503700.jpg
Аноны вопрос, последнее время не следил за новостями про РН Ангара. Что с ней? Что решили, вроде в пишут что хотят пускать с первой полезной нагрузкой в декабре 2016? То что пускают это ракета Московского производства, или уже будет Омского? или там абсолютно новое производство под нее пилят?? Расскажите в общем.
Аноним 10/01/16 Вск 16:07:07 #233 №246817 
14524312272450.jpg
>>246598
Если там черная дыра, то почему в самом центре виден свет? Ведь искривление, судя по фото, происходит на границе ЧД, а не в самом центре. В любом случае, откуда там свет по середине?
Аноним 10/01/16 Вск 16:07:35 #234 №246818 
>>246799
>беря управление на себя (саботировав?)
Ты чё, совсем?
Аноним 10/01/16 Вск 16:09:22 #235 №246820 
>>246817
А, там вместо ЧД галактика, отменяю вопрос.
Аноним 10/01/16 Вск 16:10:55 #236 №246821 
>>246818
Нюфаня?
Давай я погуглю за тебя https://2ch.hk/spc/res/151980.html
Хотя нет. В треде сам будешь искать трустори на эту тему.
Аноним 10/01/16 Вск 16:14:01 #237 №246825 
>>246818
Удачливый ублюдок. Я объебался с тредом >>246821
и нашел тебе даже пост про это. http://2ch.hk/spc/res/242910.html
Ну а если не веришь анону, то сам себе злой буратина.
sageАноним 10/01/16 Вск 17:09:35 #238 №246874 
>>246753
Но карбон не проводник. И как связаны УТ и шахта заполненная водой?
Аноним 10/01/16 Вск 18:21:05 #239 №246919 
>>246799
Скандалы, интриги, расследовая: Астронавты-саботажники НАСА.
Аноним 10/01/16 Вск 18:35:44 #240 №246928 
Если всё друг к другу притягивается, то почему тогда все планеты не упадут на солнце, и вообще всё вещество не склеится в один комок?
Аноним 10/01/16 Вск 18:40:36 #241 №246932 
>>246928
Они и падают. Все время. Просто одновременно летят друг мимо друга слишком быстро, и не успевают упасть.
Аноним 10/01/16 Вск 18:53:35 #242 №246939 
>>246874
Сажа приклеилась.
Аноним 10/01/16 Вск 19:42:15 #243 №246966 
>>246746
> Аноним 10/01/16 Вск 13:37:26 №246746
Да, окончательно.

>>246750
Но я хочу поебать мозг.
Аноним 10/01/16 Вск 21:09:21 #244 №247012 
>>246939 ок тогда

>>246874
вода для радио не очень проницаемая среда, и радиоуправление торпедами затруднено, особенно, если они далеко пилят(километры, боюсь ошибиться, но чтото было до 30км, у вмов надо проконсультироваться)
>Телеуправляемые — наведение на цель осуществляется с борта корабля или подводной лодки по проводам (оптоволокну).
Те для них делают шнурки.

Насчет карбона. Само по себе углеродное волокно проводник, те при желании можно запилить. Наса работает в этом направлении в целом, хотят проводки силовые для самолетов и прочего крафта: легче, прочнее, нет проблем перелома от вибраций, нет скинэффекта и тд, пусть и несколько более объемно. Сейчас не скину ролик(возможна не точность), но на данный момент говорят уже сделали лабораторные варианты с эквивалентными меди характеристиками, но это еще далеко от теоретического потенциала. Основная проблема, это то, что пока не умеем делать длинные волокна и основные потери на переходе между волокнами. (ну так там было заявлено).
Волокнами можно медь в принципе армировать, и собственно любые другие проводники - те вместо связующего элемента использовать не пластик в карбоне, а металлы, это если говорить о прочности.
Аноним 10/01/16 Вск 23:05:53 #245 №247072 
Все планеты крутятся с запада на восток, венера - с востока на запад. Может ли быть такое, что какая-нибудь экзопланета вообще не крутится?
Аноним 10/01/16 Вск 23:11:08 #246 №247076 
Что значит "Высокоширотная орбитальная станция"? Где проходит её орбита и правда ли, что России из неё не видно?
Аноним 10/01/16 Вск 23:39:01 #247 №247092 
>>247012
Cпасибо всё чётко написал.

А как он будут выдерживать давление, а как будут выдерживать давление стенки шахты?
Аноним 10/01/16 Вск 23:47:10 #248 №247097 
Недавно были новости о том что якобы нашли два далеких крупных транснептуновых объекта, подробностей никаких не появлялось?
Аноним 10/01/16 Вск 23:59:54 #249 №247099 
Анон, поясни, как у твердотопливных ракетных ступеней изменяют вектор тяги?
Аноним 11/01/16 Пнд 00:05:31 #250 №247108 
>>247012
А звуком низзя это дело коммуницировать, интересно?
Аноним 11/01/16 Пнд 00:13:51 #251 №247114 
>>247099

см.
>>242764
>>242765

Второй вариант -- обычное поворотное сопло с гидравликой. Это юзалось на шаттле.
Аноним 11/01/16 Пнд 15:48:44 #252 №247146 
>>247099
Чаще всего сопло качают обычной сервой.
http://www.youtube.com/watch?v=59CRsSvaCgc
https://www.dropbox.com/s/bniwav9kpxngm37/CASTOR30XL-Static-Test-Loop.wmv?dl=0 видос Castor 30XL с другого ракурса, в паблике его вроде нет, потому на дропбокс
Аноним 11/01/16 Пнд 15:50:50 #253 №247147 
Возможно ли использовать передвижные пусковые платформы для МБР, типа РТ-2ПМ2, для запуска маловесных спутников на, хотя бы, низкие орбиты?
Аноним 11/01/16 Пнд 15:55:55 #254 №247149 
>>247072
>Может ли быть такое, что какая-нибудь экзопланета вообще не крутится
Скорее всего нет. Учитывая хаотическое состояние микромира во вселенной, шарообразное тело размером с планету, имеющее нулевой момент импульса, будет такой же маловероятной флуктуацией, как и мартышка, рандомно напечатавшая таки Гамлета.
>>246928
Равновесие сил. Притяжение есть сила, уравновешенная постоянным изменением вектора скорости. Присутствие этого постоянного изменения вектора характерно для кругового движения. Это изменение вектора в свою очередь порождает силу, которую для наглядности можно назвать "центробежной". Но сам термин фиктивен, по факту эта сила является следствием инерции, исходящим из наличия центростремительной силы. Которая в свою очередь пусть и не является фиктивной, но является производной, рожденной в случае орбитального движения космических тел, как раз таки гравитационной силой притяжения.
Если попробовать попроще, с очевидным наземным экспериментом - раскручиваемым на нити грузом: наличие массы груза создает инерцию при движении, которая при равной скорости стремится по первому закону Ньютона полететь прямолинейно по приданному вектору скорости. Однако прямо ей не дает лететь нить, которая, создавая силу натяжения, тут же рождает центростремительное ускорение, вектор которого направлен к центру окружности вращения. И тут казалось бы косяк, потому что и сила центростремительная и сила натяжения нити направлены к центру, имеют ненулевое положительное значение, но в результате дают ноль в сумме, ведь силы находятся в равновесии. И тогда, не готовым к нетривиальному матану умам на помощь приходит фиктивная "центробежная" сила, которая в одном своем названии несет очевидный смысл и объясняется первым законом Ньютона.
Собственно математическое решение этому казусу дано, но заключается оно в массивных расчетах совмещения инерциальных и неинерциальных систем отсчета, с множественными матрицами поворота, поэтому проще рассказывать про нитки и первый закон. В случае со звездами гравитация исполняет роль нити, сила притяжения аналогична силе натяжения нити.
Аноним 11/01/16 Пнд 16:03:04 #255 №247151 
>>247147
Да,
Аноним 11/01/16 Пнд 16:08:45 #256 №247153 
https://www.youtube.com/watch?v=WoiVej1rccs
Уже в 60-х годах был практически рабочий движок с 850 сек импульса, был задел на дальнейшее повышение за 1000 сек, если сейчас кто-то реально хочет лететь на Марс, почему не реанимируют проект а собираются что-то там пилить на химических перделках? Они не понимают, что это практически невозможно или просто не собираются никуда лететь?
Аноним 11/01/16 Пнд 16:13:03 #257 №247155 
>>247147
Только если
1) спутнику похуй на нештатные нагрузки (чего практически никогда не бывает, разве что это калибровочная сфера для радара или что-то в этом духе), а также он бесшовно интегрирован с МБР вместо БЧ
2) МБР способна вообще работать в таком режиме без допила (со штатной СУ и т.д.), о чем надо спрашивать в /wm/, они там больше в курсе.

Так-то МБР и конверсионная РН на её базе это не одно и то же, их обычно допиливают - рулевые движки, лишняя ступень/РБ, удлинненный обтекатель, адаптер для ПН и т.д. Проще из ШПУ хуйнуть, чем заморачиваться с интеграцией этой ебалы еще и с подвижной платформой.
Аноним 11/01/16 Пнд 16:23:47 #258 №247156 
>>247153
Потому что
1) практически рабочий != рабочий (можешь например у пейсх спросить, они объяснят), не вполне была ясна термодинамика этой хуиты в вакууме, не вполне был ясен вопрос пассивации, и т.п., в общем не летало = не существует, как и всегда в аэроспейсе.
2) есть последствия для верхней атмосферы (засоряется изотопами), да собственно где ни запусти - везде будут проблемы в том или ином виде
3) есть опасность проеба РН с этой хуитой
4) есть опасность для экипажа
5) существуют малоимпульсные траектории до Марса для ионников, которые избавляют от необходимости городить огород с этой мечтой доктора Стрейнджлава за миллионы денег. Собственно НАСА в текущем варианте так и собирается железо туда таскать, а вовсе не на химии.
Аноним 11/01/16 Пнд 16:28:10 #259 №247158 
>>247153
Вот еще было программа https://en.wikipedia.org/wiki/Project_Timberwind
Аноним 11/01/16 Пнд 17:24:37 #260 №247180 
>>247158
> ПРО с ядерным движком
наркомания какая-то, нахуя, какие профиты
Аноним 11/01/16 Пнд 20:50:04 #261 №247229 
Не вижу в ПК версии стеллариума искусственных спутников, отображение включил. На ведройде все видно. Алсо, спутники онлайн показывает?
Аноним 11/01/16 Пнд 20:53:53 #262 №247230 
>>245254 (OP)
Может ли душа человека покинуть черную дыру?
Аноним 11/01/16 Пнд 21:02:19 #263 №247233 
>>247230
В этом разделе нет души.
Аноним 11/01/16 Пнд 21:05:05 #264 №247234 
>>247230
Душа имеет вес, следовательно материальна, следовательно не может
Аноним 11/01/16 Пнд 21:05:24 #265 №247235 
>>247230
Нет
Аноним 11/01/16 Пнд 21:06:19 #266 №247236 
>>247229
>Не вижу в ПК версии стеллариума искусственных спутников
У меня отображаются.
Аноним 11/01/16 Пнд 21:08:20 #267 №247237 
>>247234
Есть мнение что души после смерти туда и попадают
Аноним 11/01/16 Пнд 21:55:10 #268 №247245 
Почему бы ракеты не разгонять маглевом до скорости звука?
Аноним 11/01/16 Пнд 22:21:55 #269 №247261 
>>247245
> Почему бы ракеты не разгонять маглевом до скорости звука?
Мизерный выигрыш за счет большой-большой ебли.
Аноним 11/01/16 Пнд 22:33:53 #270 №247276 
>>247261
>Мизерный выигрыш
Due to factors including the exponential nature of the rocket equation and higher propulsive efficiency than if a rocket takes off stationary, a NASA Maglifter study estimated that a 270 m/s (600 mph) launch of an ELV rocket from a 3000-meter altitude mountain peak could increase payload to LEO by 80% compared to the same rocket from a conventional launch pad.[5]
Аноним 11/01/16 Пнд 22:39:20 #271 №247277 
>>247276
А теперь представь, как будет выглядеть эта штука, и не забывай, что запускать надо почти вертикально, ебануть с пологого трамплина не выйдет. Хотя с "мизерным" я погорячился.
Аноним 11/01/16 Пнд 22:42:35 #272 №247279 
>>247277
>запускать надо почти вертикально
прямые рельсы постепенно идут в пологую гору и к вершине достигают угла в 45 градусов
Аноним 11/01/16 Пнд 22:44:37 #273 №247280 
>>247279
> и к вершине достигают угла в 45 градусов
И ракета тратит на вертикализацию треть выигранной дельты?
Аноним 11/01/16 Пнд 22:52:43 #274 №247284 
>>247280
Так конец пути на высоте 3 км, там уже поворачивать пора
Аноним 11/01/16 Пнд 22:56:24 #275 №247286 
>>247284
В норме там максимум градусов десять от вертикали при обычном запуске.
Аноним 12/01/16 Втр 00:40:04 #276 №247327 
>>246609
Это ты сейчас так коротационный круг описал? У нас на районе за такие описания убивают нахер.
Аноним 12/01/16 Втр 00:42:17 #277 №247329 
>>246611
Предполагается, что на Европе существуют недавно закрывшиеся проруби с толщиной льда всего несколько десятков метров (через которые и течет всякий кал и формирует молодую бескратерную поверхность).
Аноним 12/01/16 Втр 00:46:20 #278 №247331 
>>247076
Это значит что орбита поближе к полярной, чем у нынешней. России не видно из нынешней, а точнее видно, но в основном сбоку (орбита ближе к экваториальной).
Аноним 12/01/16 Втр 00:50:02 #279 №247338 
>>247180
Звездные войны же, тогда для обороны ничего не жалели, ни свой бюджет, ни бюджет потенциального противника, ни здравый смысл обоих.
Аноним 12/01/16 Втр 00:51:38 #280 №247339 
>>247180
Емнип, один из вариантов проекта "Орион" предусматривал тысячетонную ракету на взрывном атомном двигателе оснащенную гигатонным термоядерным зарядом
Аноним 12/01/16 Втр 00:52:44 #281 №247341 
>>247153
Тогда оказалось беззадач в силу отсутствия больших ПН. Движок таки практически рабочий, не летал. Сейчас это куда сложнее в силу радиофобии (а такая ебалайка, ебнувшись на середине отработки, может очень нехороших изотопов выдать, это вам не безобидная ядерная бомба).
мимобиофизик
Аноним 12/01/16 Втр 00:55:46 #282 №247344 
>>247339
Это, видимо, малый Орион.
Аноним 12/01/16 Втр 00:58:01 #283 №247345 
>>247344
Ну возможно.
Аноним 12/01/16 Втр 00:59:06 #284 №247346 
>>247344
Немножко ошибся
"Военные космические корабли на базе «Ориона»
«Orion» battleship — инициативный концепт ядерно-импульсного боевого космического корабля, предложенный Фриманом Дайсоном в начале 1960-х. Предназначался для завоевания господства на околоземных орбитах и в межпланетном пространстве и нанесения ударов по объектам на поверхности Земли и других небесных тел. Не рассматривался всерьёз военными и не вышел за стадию эскизов.
Doomsday Orion — концепт сверхтяжёлой ядерно-импульсной баллистической ракеты, предложенной ВВС США, способной доставить к цели 1600-тонный термоядерный заряд мощностью свыше 3 гигатонн. Не получил развития."
Аноним 12/01/16 Втр 08:20:25 #285 №247383 
Можно ли теоретически выходить из "Союза" в открытый космос?
Аноним 12/01/16 Втр 08:22:33 #286 №247384 
>>247383
>теоретически
Можно, но только один раз
Аноним 12/01/16 Втр 08:26:10 #287 №247386 
>>247384
А зайти?
Аноним 12/01/16 Втр 08:26:49 #288 №247387 
>>247339
Орион это РН. А тут система ПРО. На ебаном ядерном движке.
Аноним 12/01/16 Втр 08:28:55 #289 №247388 
>>247279
И ракету вместе с ПН плющит от поворота сверх g.
Аноним 12/01/16 Втр 08:30:38 #290 №247389 
>>246604
>Вопрос скорее в зависимости нынешних орбит от гравитации в текущем секторе галактики. Сильно ли эта гравитация влияет на стабильность орбит?
Нельзя влиять на то, что неизвестно есть ли вообще. Никто не знает, стабильна ли СС вообще.
Аноним 12/01/16 Втр 11:50:41 #291 №247407 
>For instance, when two atoms of hydrogen com­bine to form one molecule of H2, some 100 kilocalories of energy are
released per two-gram mole. This means that a 50-50 (by weight) mixture of monatomic hydrogen and ordinary hydrogen would have a performance of some 1000-1100 seconds.
Почему ещё не пилим?
Аноним 12/01/16 Втр 11:57:58 #292 №247408 
>>247383
Можно. Даже из шарика Восхода выходили (с помощью складного шлюза), а у Союза вообще шлюзуемый орбитальный отсек для этого есть.
https://ru.wikipedia.org/wiki/%D0%A1%D0%BE%D1%8E%D0%B7-4
Аноним 12/01/16 Втр 13:19:42 #293 №247419 
Тут, конечно, сидят ракующие уёбки в основном, из-за которых доска скатилась в говно, но таки спрошу, мало ли.
Нужна книга "Planetary landers and entry probes", где спиздить?
Аноним 12/01/16 Втр 13:21:19 #294 №247421 
>>247408
Только смысла в этом мало. Американский подход всяко лучше, а шлюз на "Союзе" - просто следствие появившегося "лишнего" отсека (который есть костыль).
Аноним 12/01/16 Втр 13:27:04 #295 №247423 
>>247419
Вбил в гугол, получил http://www.e-reading.club/bookreader.php/138786/Ball_-_Planetary_Landers_and_Entry_Probes.pdf
Аноним 12/01/16 Втр 13:35:09 #296 №247426 
14525949091450.jpg
>>247419
>я вас презираю, но помогите
Аноним 12/01/16 Втр 13:38:47 #297 №247427 
>>247421
Разгерметизировать ВЕСЬ КОРАБЛЬ лучше, чем иметь шлюз?
Хуй знает. Союз может и костыльный, но наличие орбитального отсека - хорошее решение. И отдохнуть на диванчике можно кстати, зачем он там? и выйти в космос, а спускаемый модуль остается мелким и надежным. Не то, что бы Аполлон, Орион, Дрэгон или ППТК были хуже в этом плане, но дополнительный сбрасываемый отсек - тоже годно, по крайней мере не делает необходимым разгерметизацию всего корабля, нивелируя необходимость усложнять некритическую электронику и прочие компоненты, делая их работоспособными в вакууме. Уверен, что критичные компоненты и так должны быть способны на это в случае незапланированной разгерметизации.
Аноним 12/01/16 Втр 13:42:50 #298 №247429 
Почему спасали ТТРД шаттла? Я понимаю, зачем хотели спасать сам челнок - такая-то йоба, да и движки дорогие. Понимаю, зачем планировали спасение первых ступеней Энергии - крутые дорогие РД-170.
А пороховую сраную бочку, падающая в океан нахуй спасать? Все равно большая часть - это топливо, которое и так по-новой закладывать, а остальное - люминь по краям топлива и чутка электроники на выброс и какая-то рудиментарная система управления вектором тяги. Проще с нуля собрать заново бочку из люминя, да набить рассыпухой и топливом, чем ебаться с коррозией и прочим говном. Вон, Челленджеру припекло от криво собранной бочки тогда.
Аноним 12/01/16 Втр 13:45:52 #299 №247430 
>>247427
Встречал мнение, что шлюзы начали городить как раз от того, что не смогли в электронику, которая спокойно переносила бы разгерметизацию.
Аноним 12/01/16 Втр 13:52:38 #300 №247433 
>>247430
Как вариант, хорошее объяснение.
Тем не менее, вакуумная электроника дороже и возможно тяжелее обычной космической, так что решение имеет и положительные стороны, в том числе и описанные мною выше.
Аноним 12/01/16 Втр 14:05:40 #301 №247434 
>>247429
Быдло то надо было успокоить. Мол смотрите, эта еба совсем не дорохая. Мы же вон ее по джва раза используем.
Аноним 12/01/16 Втр 14:13:03 #302 №247436 
>>247419
Я бы рассказал тебе про libgen, но лучше иди в /b/, ведь все равно разницы нет
Аноним 12/01/16 Втр 16:38:58 #303 №247462 
>>247427
>И отдохнуть на диванчике можно кстати, зачем он там? и выйти в космос
А главное посрать в одиночестве.
Аноним 12/01/16 Втр 17:00:38 #304 №247464 
Что если бы бабушка стала дедушкой и Солнце вдруг превысило четвертую космическую? Повлияло бы это хоть как-то на дальнейшее развитие жизни внутри Солнечной Системы в долгосрочной перспективе?
Аноним 12/01/16 Втр 17:02:23 #305 №247465 
>>247464
А ты очень оригинален, уже почти сто постов не было этого вопроса>>246600
Аноним 12/01/16 Втр 17:05:27 #306 №247466 
Возможно ли где-то во Вселенной существование планеты с разумными огурцами разрабатывающими космическую программу?
Аноним 12/01/16 Втр 17:19:01 #307 №247469 
>>247466
да.
Аноним 12/01/16 Втр 18:36:10 #308 №247498 
>>247469
Благодарю
Аноним 12/01/16 Втр 18:49:12 #309 №247510 
>>247466
Мультивселенная. Все возможно.
Аноним 12/01/16 Втр 19:40:03 #310 №247540 
14526168032950.png
>В этой ситуации время полёта в земной системе отсчёта составит примерно 12 лет, тогда как по часам на корабле пройдёт 7,3 года
Увидел в википедии про полёты к звёздам вот такое. В статье о межзвёздных перелётах.
Вопрос: есть фактическое подтверждение разного течения времени при уже осуществлённых полётах?
Как ни верти а на Луну и на орбиту летали. Скорости конечно были очень далеки от световых, но и часы могут быть точные. Была ли установлена разница по часам внутри полётного модуля и часах на земле? То есть вот документально чтобы?
Аноним 12/01/16 Втр 19:40:45 #311 №247541 
>>247540
Бамп интересному вопросу
Аноним 12/01/16 Втр 19:49:34 #312 №247545 
>>247540
>Была ли установлена разница по часам внутри полётного модуля и часах на земле? То есть вот документально чтобы?
Эксперимент Хафеле — Китинга, например.
Аноним 12/01/16 Втр 19:51:39 #313 №247546 
14526174992580.png
Еще в школе всем рассказывали, что в вакууме и перо, и чугунная гиря будут падать на землю с одинаковой скоростью. Отсюда делается вывод, что тела с любой массой притягиваются с одинаковой скоростью
Тогда почему они, в свою очередь, не притягивают землю к себе? И вообще это все очень странно.
Аноним 12/01/16 Втр 19:54:45 #314 №247547 
>>247546
>почему они, в свою очередь, не притягивают землю к себе?
Как это не притягивают, когда притягивают. Только масса пера, или гири, по сравнению с Землей вообще ни о чем.
Аноним 12/01/16 Втр 19:57:29 #315 №247549 
>>247547
>Только масса пера, или гири, по сравнению с Землей вообще ни о чем.
Но ведь все тела одинаково притягиваются, независимо от массы. Пруф: эксперимент с пером и гирей. Землю не должно волновать какая там масса у пера.
Аноним 12/01/16 Втр 20:14:02 #316 №247558 
>>247540
Естественно. Любая орбитальная хуита, что нуждается в особо точном позиционировании (пространственном и временном) и имеет бортовой источник точного времени - разработана с учетом поправок на это. Начиная от навигации и заканчивая большинством научных нагрузок вроде гравиметрии или телескопов (у того же Спектра-Р, которому нужно позиционирование в районе сантиметров и особо точный отсчет времени, т.к. сигнал от него нужно коррелировать с наземными телескопами, составляющими второе плечо Радиоастрона)
Аноним 12/01/16 Втр 20:18:35 #317 №247567 
14526191160560.png
>>247549
>Но ведь все тела одинаково притягиваются, независимо от массы.
Да что ты!? Этож охуеть просто.

Пиздос, это же в 5-м классе проходят.
Аноним 12/01/16 Втр 20:22:26 #318 №247569 
>>247567
Тогда почему гиря и перо падают в вакууме с одинаковой скоростью?
Аноним 12/01/16 Втр 20:29:16 #319 №247575 
>>247569
Во-первых с одинаковым ускорением, а не скоростью, во-вторых обрати внимание КУДА они падают. Ускорение свободного падения на Земле, Марсе, Луне, Юпитере очень разные. Мешок с требухой типа тебя на этих планетах будет падать с разным ускорением, при том, что твоя масса будет ровно той же самой на всех планетах.
Аноним 12/01/16 Втр 20:32:01 #320 №247579 
>>247569
> Тогда почему гиря и перо падают в вакууме с одинаковой скоростью?
Потому что это такое упрощение, разница ускорения там проявится в хуйнадцатом разряде.
Аноним 12/01/16 Втр 20:39:46 #321 №247582 
>>247575
>>247579
Так, я не понял. Получается, если одновременно кинуть на Землю, например, Солнце и перо, то они тоже упадут с одинаковым ускорением? А Солнце, в свою очередь, будет притягивать к себе Землю. Тогда почему перо не притягивает Землю, оно же притягивается к Земле с одинаковым ускорением вместе с Солнцем.
Аноним 12/01/16 Втр 20:49:31 #322 №247588 
>>247582
> Получается, если одновременно кинуть на Землю, например, Солнце и перо, то они тоже упадут с одинаковым ускорением?
Нет. Одинаковое ускорение это миф просто округление, которое будет работать, когда масса одного из тел пренебрежительно мала. В сравнении с массой Луны, например, масса пера и кувалды будет одинаково пренебрежительно мала. Масса Солнца и пера - не одинаково.
> Тогда почему перо не притягивает Землю, оно же притягивается к Земле с одинаковым ускорением вместе с Солнцем.
Оно притягивает. Силу эту можешь самостоятельно посчитать по формуле из поста >>247567 , эта сила, если я сейчас не слишком туплю, будет действовать одинаково на оба тела, но если для массы пера этасила даст ускорение в 1 единицу, то для массы Земли - в 0 целых нихуя десятых.
Аноним 12/01/16 Втр 20:53:16 #323 №247592 
Космической музычки в тред. Снято на МКС.


https://www.youtube.com/watch?v=KaOC9danxNo
Аноним 12/01/16 Втр 20:54:33 #324 №247594 
Возможно ли разрушить или как то повредить чд воздействием из вне? А нейтронную звезду? Какие теперь предположения на счет происхождения смчд и квазаров?
Пожалуйста, Анон. А то не усну.
Аноним 12/01/16 Втр 20:57:28 #325 №247599 
>>247594
>А нейтронную звезду?
А какие проблемы с повреждением НЗ?
Аноним 12/01/16 Втр 21:01:52 #326 №247604 
>>247594
>повредить чд
Только если кофе прольешь на записки с решением уравнений ОТО.
Вы заебали со своей боязнью Черных дыр. Это ебаная теория поехавших математиков и физиков. Никаких эксперементальных подтверждений нет.
Аноним 12/01/16 Втр 21:02:18 #327 №247605 
>>247594
>как то повредить чд воздействием из вне
Можно сделать ЧБ более упитанной, воздействием извне. Больше ничего.
>А нейтронную звезду?
Нейтронные звезды в тесных системах сталкиваются, как любые другие объекты.
>Какие теперь
Такие же, как и раньше.
Аноним 12/01/16 Втр 21:04:23 #328 №247607 
>>247549
все правильно, после того как ты перестаешь разнимать землю и перо, перо начинает притягивать землю и та, как послушная своему хозяину собачка, устремляется к перу.
Потом они сливаются и закат
the end
Аноним 12/01/16 Втр 21:07:40 #329 №247608 
>>247607
Таким образом, можно смещать орбитальное положение земли постоянным бросанием перьев. Гениально!
Аноним 12/01/16 Втр 21:11:24 #330 №247612 
>>247608
Можно, только очень долго и где бы найти достаточной величины птицеферму.
Аноним 12/01/16 Втр 21:11:44 #331 №247613 
>>247605
Тогда что происходит при слиянии двух чд? Меньшая разве не разрушается? Как такое возможно?
Аноним 12/01/16 Втр 21:13:00 #332 №247614 
>>247540
Воу, паринь, ты действительно недалек в вопросах физики. Если ты ознакомился по совету этого>>247545 многоуважаемого дона с информацией, то уже знаешь, что для экспериментального подтверждения релятивистского замедления достаточно авиационных скоростей.
Аноним 12/01/16 Втр 21:15:52 #333 №247615 
тред не читал
Аноним 12/01/16 Втр 21:16:50 #334 №247617 
>>245254 (OP)
Ах, ладно. Задам вопрос.

Как известно, что ресурсы на Земле ограничены. Значит ли это, что через каких-нибудь 300 лет если человечество не освоит космос, то нас ждет падение в средневековую парашу? Ну закончится уран, нефть, газ, уголь! Что дальше? Альтернативные источники энергии не прокатят. Они не смогут обеспечить столько населения. В конце концов сам кислород не вечен. Земля замедляет вращение. Ответь, анон. Значит ли это, что без покорения космоса нам в итоге придет тотальный пиздец уже в этом тысячилетии?
Аноним 12/01/16 Втр 21:23:30 #335 №247619 
14526230103760.webm
>>247613
>что происходит при слиянии двух чд
Они сливаются в одну. Из двух ЧД с массами М1 и М2 Получается ЧД массой М1+М2.

>Как такое возможно?
А что тебя смущает?
Аноним 12/01/16 Втр 21:33:36 #336 №247623 
14526236170600.jpg
>>247594
>Возможно ли разрушить или как то повредить чд воздействием из вне?
Предполагается, что в будущем все ЧД разрушатся - когда скорость расширения пространства станет достаточно велика. Это можно назвать внешним воздействием.

Так что вот тебе способ: узнай, почему растет постоянная Хаббла, найди способ влиять на ее изменения, и измени её в небольшой области вокруг сферы Шварцшильда - черной дыры. Если скорость расширения пространства в окрестностях будет достаточно велика, дыра "взорвется".
Аноним 12/01/16 Втр 21:35:09 #337 №247624 
>>247599
>А какие проблемы с повреждением НЗ?
Предложи способ разделить ее на части.
Аноним 12/01/16 Втр 21:39:16 #338 №247626 
>>247617
>Значит ли это, что через каких-нибудь 300 лет если человечество не освоит космос, то нас ждет падение в средневековую парашу?
Нет, не значит.
>Закончится нефть, газ, уголь!
Ученые не однозначны в вопросе закончится ли вообще, а если закончится, то когда.
>закончится уран
Вообще пушка. Реакторы на быстрых нейтронах уже готовы, они могут работать на MOX-топливе, то есть буквально на отходах. Определенные надежды возлагаются на ИТЭР. Альтернативные источники энергии стремительно расширяют свою долю, они может и не заменят традиционные совсем, но тенденция довольно позитивная.
>В конце концов сам кислород не вечен.
А что с ним? Если человек не вырубит все леса и не убъет весь планктон, то с кислородом ничего не станет.
>Земля замедляет вращение
Земля замедляет вращение уже 4.5 млрд лет, все остановиться не может. Последнее о чем вообще стоит волноваться.
>Значит ли это, что без покорения космоса
Покорения в каком виде? Расселение по планетам и спутникам планет, что бы каждая из колоний была на полном самообслуживании? Я не берусь давать оценки развитию космонавтики на 1000 лет. Но и ожидать ближайшего пиздеца в 300 лет тоже не стал бы.
Аноним 12/01/16 Втр 21:39:47 #339 №247627 
>>247617
Очевидно что лишние человеки вымрут или что более вероятно их поубивают другие человеки.
Аноним 12/01/16 Втр 21:42:41 #340 №247629 
>>247624
Уебать одну в другую и получить немного золота, например.
https://www.cfa.harvard.edu/news/2013-19
Аноним 12/01/16 Втр 21:44:57 #341 №247632 
Подскажите-ка мне грав модель земли и луны, поточнее чем шарик
Аноним 12/01/16 Втр 21:45:08 #342 №247633 
>>247627
Вот этого двачую. Там уже нашли каких-то паскудных микробов, которые антибиотики на хую вертели. Будет мор, войны и Рагнарек скорее, чем человечество останется без энергии в каком-либо виде.
Аноним 12/01/16 Втр 21:52:39 #343 №247636 
>>247632
Гугли:

1. GRACE по Земле
http://www.csr.utexas.edu/grace/gravity/

2. GRAIL по Луне
https://svs.gsfc.nasa.gov/cgi-bin/details.cgi?aid=4014
Аноним 12/01/16 Втр 21:53:55 #344 №247638 
14526248358890.gif
>>247608
А гравитационные маневры понемногу замедляют Юпитер, так-то.
Аноним 12/01/16 Втр 21:54:05 #345 №247639 
>>247623
Кем предполагается? Каким образом расширение пространства в окрестностях ЧД должно ее разрушить?
Аноним 12/01/16 Втр 21:56:32 #346 №247643 
>>247638
Ох, долетаются. Мало того что все небо своими ракетами издырявили, так еще и Юпитер на солнце уронят.
Аноним 12/01/16 Втр 21:58:55 #347 №247644 
>>247638
Поназапускают своих Вояджеров, а Юпитер потом колом встает и вертеться отказывается.
Аноним 12/01/16 Втр 22:06:29 #348 №247646 
>>247639
>Кем предполагается?
Научным сообществом. Одна из теорий эволюции вселенной подразумевает вариант "большого разрыва".
>Каким образом расширение пространства в окрестностях ЧД должно ее разрушить?
Ты немного узко воспринимаешь модель расширяющейся вселенной.
Аноним 12/01/16 Втр 22:22:00 #349 №247653 
>>247629
Уж лучше за алмазами на Сатурн
Аноним 12/01/16 Втр 22:27:22 #350 №247655 
>>247653
Лучше сидеть на жопе и из комнаты не выходить. Космос - место опасное.
Аноним 12/01/16 Втр 23:03:14 #351 №247668 
>>247646
Ты не ответил ни на первый, ни на второй вопрос.
Придется отвечать самому себе.

>Кем предполагается? Каким образом расширение пространства в окрестностях ЧД должно ее разрушить?
Аккреция темной энергии на черную дыру вызывает сокращение массы ЧД.
http://arxiv.org/abs/astro-ph/0505618

Чем меньше масса, тем быстрее ЧД испаряется.
Аноним 12/01/16 Втр 23:13:24 #352 №247673 
>>247636
о спасибо
Аноним 12/01/16 Втр 23:17:09 #353 №247674 
>>247668
Тут два человека отвечают. Имелось в виду не сокращение массы ЧД через туннельный эффект и флуктуации физвакуума, а непосредственный разрыв горизонта-сферы. Если пространство будет расширяться быстрее скорости распространения гравитации, то изменится топология: сферу "развернет" в плоскость, масса не сможет удержать горизонт.
Аноним 12/01/16 Втр 23:55:02 #354 №247680 
>>247674
>разрыв горизонта-сферы
А почему, собственно, горизонт должен быть сферическим? Может, там точка, на самом деле.
Аноним 13/01/16 Срд 00:00:10 #355 №247682 
>>247674
Мысль понятна.
Аноним 13/01/16 Срд 00:20:53 #356 №247683 
Я правильно понял, что пилотируемые полеты на орбиту в данный момент совершаются только на Союзах? Если так, то какие пилотируемые космические корабли находятся в разработке и, если таковые имеются, какие у них преимущества перед Союзом?
Аноним 13/01/16 Срд 00:58:12 #357 №247689 
>>247683
Китайцы же летают на "как бы не союзах" еще.
У американцев сразу три - Дракон2 от маскика нашего любимого и ЦСТ-100 от боинга. Задачи, в принципе, как у Союза - маршрутка до НОО (конкретно до МКС), хотя Маск вроде пытается натянуть дракона на глобус и расширить сферу применения. Еще Орион от НАСА - вот там уже все серьезнее, чем у Союза, это летать далеко за пределы НОО (первая толковая миссия вроде как к редиректнутому на орбиту Луны камню, потом марсианские миссии на нем будут делать).
У нас еще постоянно что-то в разработке, но конкретику назвать не могу - очень уж планы постоянно меняются, не уследишь.
По остальным не знаю.
Аноним 13/01/16 Срд 01:10:58 #358 №247691 
14526366582190.jpg
>>247689
У роскосого ППТС же.
Аноним 13/01/16 Срд 01:19:25 #359 №247695 
>>247691
А вот и нет
Аноним 13/01/16 Срд 01:32:50 #360 №247696 
>>247695
Обоснуй.
Аноним 13/01/16 Срд 01:35:03 #361 №247697 
>>247695
Ват?

Олсо, MPCV вот до пиздеца насовский корабль. Консервативный очень. Говорят, что там охуенные технологии по работе с алюминием юзаются, но меня всё-таки мучает вопрос: неужели из композитов было бы хуже? Даж в РФ капсула ППТС из них, уверен, что техническая возможность у подрядчиков насы была.
Аноним 13/01/16 Срд 01:37:34 #362 №247699 
>>247697
они вроде и то и другое пилят, композит просто запасной вариант пока
если я правильно понял о чем ты, была вроде насовская агитка, если надо найду
Аноним 13/01/16 Срд 01:49:52 #363 №247704 
>>247699
Два сразу? Ясно, значит нихуя.
Давай, если не лень. Просто исхожу из того, что композит в любом случае будет всяко лучше, и кроме как консерватизмом не могу оправдать алюминий.
Аноним 13/01/16 Срд 08:28:50 #364 №247734 
14526629307310.jpg
>>245254 (OP)
Если сбросить юпитер на солнце, повлияет ли это на солнечную активность? Или солнце зохавит эту хуйню и не заметит?
Аноним 13/01/16 Срд 08:43:26 #365 №247735 
>>247734
Да, скорее всего для нас такое событие повлечет как минимум неприятные последствия. Не говоря уже о последствиях пропажи Юпитера с орбиты.
Аноним 13/01/16 Срд 10:21:52 #366 №247747 
>>247734
Повлияет, но всем будет похуй. Протуберанец пожирнее вылетит, магнитное поле всколыхнется, пару спутников наебнется. Юпитер в тысячу раз меньше, чем Солнце, но нереально вообразить лобовое столкновение таких масштабов, он будет по касательной лететь, его распидорасит приливными силами и метаном оросит солнце.

>>247735
Для жизни на земле катаклизма не будет. Для нашей цивилизации - возможно. Спутники повырубает и электросети посжигает.
Аноним 13/01/16 Срд 11:54:37 #367 №247769 
>>247735
>Не говоря уже о последствиях пропажи Юпитера с орбиты.
Мало самого факта пропажи, когда этот жирный пидорас пойдет со своей орбыты через внутреннюю часть солнечной системы охуеют все. И падение на Солнце будет полной хуетой, по сравнению с этим.
Аноним 13/01/16 Срд 12:46:36 #368 №247780 
Сап, /spc
На Луне астронавтам удобнее всего было передвигаться своеобразными прыжками. А как будет удобно ходить на Марсе?
Аноним 13/01/16 Срд 12:53:52 #369 №247781 
>>247780
придется медленно ползать, чтобы не утонуть в зыбучих песках.
Аноним 13/01/16 Срд 12:57:41 #370 №247782 
>>247780
Ответа не знаю, но вот тебе ускорения свободного падения Земли, Луны и Марса для сравнения: 9,8 ; 1,6 ; 3,7. Лично я думаю, что легче будет ходить.
Аноним 13/01/16 Срд 13:10:39 #371 №247785 
>>247780
Скафандр, со всем говном как раз добавит массы для удобного хождения, как на Земле.
Аноним 13/01/16 Срд 13:11:28 #372 №247786 
>>247769
>И падение на Солнце будет полной хуетой
я бы не стал так утверждать. Тут обычные вспышки мониторят, при нормальной работе солнца, а тут такой кусок, шлеп на поверхность.
До ядра то эта хуйня через миллионы лет дойдетхз через сколько.
Те сравнивать всю массу юпитера со всей массой солнца бессмысленно и то что она всего 0.1 процента - совсем не ответ на вопрос что будет.
Вся масса фотосферы меньше массы юпитера (скорее всего)
Также зависит конечно как упадет, но допустим начнет орбитить в этой фотосфере, там же содомия начнется, локальная, вместо работы по переизлучению и переработке внутреннего спектра излучения. А это внутреннее излучение будет хуячить напрямую, а у нас там что ?
Если Юпитер просто впилится, то даже в этом случае он достаточно долго будет там плавать ледышкой, хз к каким эффектам это приведёт.
Аноним 13/01/16 Срд 13:14:08 #373 №247787 
>>247782
>>247785
Я тоже так думал. Спасибо, аноны
Аноним 13/01/16 Срд 13:15:19 #374 №247788 
добавлю
>>247786
> хз к каким эффектам это приведёт.
к примеру к северным сияниям на экваторе, вот негры то охуеют
и очень может быть что они единственные кто сможет это к тому моменту сделать
Аноним 13/01/16 Срд 13:24:26 #375 №247793 
14526806667790.gif
>>247786
Всех тобой описанных эффектов ты уже может и не увидишь, потому что Юпитер раскидает Пояс астероидов и ты помрешь, как сраный динозавр.
Аноним 13/01/16 Срд 13:33:41 #376 №247796 
>>247793
Вопрос говно изначально, и по этой причине я его и не рассматривал.
Я просто решил обратить внимание, того анона который так пренебрежительно отозвался о ситуации падения юпитера на солнце, на тот факт, что солнце это не кусок манной каши, равномерно перемашенный с желтым теплом и вкусняшками, как ему по всей видимости представляется.
А достаточно сложный объект, находящияся в динамическом равновесии, имеющий свою дифференцированную структуру, в том числе и с нежными частями.
Аноним 13/01/16 Срд 13:35:19 #377 №247797 
14526813193610.jpg
>>247796
>солнце это не кусок манной каши, равномерно перемашенный с желтым теплом и вкусняшками
МММ...
Аноним 13/01/16 Срд 15:20:51 #378 №247817 
>>245254 (OP)
Слышал, что Луна падает на Землю на 2см в год, это правда? И почему так? Ей же не о что тормозить.
Аноним 13/01/16 Срд 15:29:56 #379 №247822 
>>247817
Среднее расстояние между Луной и Землей постепенно увеличивается, а не уменьшается. Происходит это из-за замедления вращения Земли в результате приливной гравитации Луны. То есть Земля теряет момент вращения, который "переходит" к Луне и поднимает ее орбиту.
Аноним 13/01/16 Срд 16:24:06 #380 №247828 
>>247817
Да, этот >>247822 прав, хуй знает откуда ты высрал уменьшение, луна удаляется.
Остальные спутники падают по ряду причин - торможение о верхние слои атмосферы, приливные силы, хуйня, малафья.
Вот Фобос как раз близится к Марсу со скоростью 2см/год и через 30-50 мегалет его распидорасит либо приливными силами, либо о поверхность.
Аноним 13/01/16 Срд 16:26:21 #381 №247830 
>>247828
>хуй знает откуда ты высрал уменьшение

Перепутал. Однако спасибо за ответы.
Аноним 13/01/16 Срд 16:27:36 #382 №247831 
>>247828
> Вот Фобос как раз близится к Марсу со скоростью 2см/год и через 30-50 мегалет его распидорасит либо приливными силами, либо о поверхность.
Заебись, где купить билет на первый ряд?
Аноним 13/01/16 Срд 16:57:25 #383 №247847 
>>247831
Я полагаю, что к тому моменту мы сами сможем наблюдать аналогичное падение и на сраную земляшку, если ничего не будем предпринимать, или не выпилимся к чертям.
Я вот не собираюсь помирать, завожу сраный челнок и сваливаю со сраной земляшки, буду тусить по галактике, пока большой разрыв пукана не настигнет, или не придумаю, как его забороть или обойти.
sageАноним 13/01/16 Срд 18:02:03 #384 №247849 
>>247847
>К тому моменту
Никого уже не будет, и Земли не будет, а если будет, то на ней давно ничего не будет.
Аноним 13/01/16 Срд 18:02:59 #385 №247850 
>>247849
Ты так уверенно это говоришь? Ты экстрасенс?
Аноним 13/01/16 Срд 18:05:12 #386 №247851 
>>247619
Сливаются это значит по крайней мере одна из них должна изчезнуть т.е. разрушиться. Какие такие силы могут это сделать?
Аноним 13/01/16 Срд 18:16:32 #387 №247854 
>>247831
у доктора кто спроси, у него есть, инфа сотка
sageАноним 13/01/16 Срд 18:17:36 #388 №247855 
>>247850
Пиздуй читать звездную эволюцию. Базарю, еще захочешь.
Аноним 13/01/16 Срд 18:18:45 #389 №247856 
>>247851
>т.е. разрушиться
В полстакана водки налили стакан водки, значит ли это, что полстакана водки разрушилось?
Аноним 13/01/16 Срд 18:20:34 #390 №247857 
>>247856
Сравнение некорректно.
Аноним 13/01/16 Срд 18:21:45 #391 №247859 
>>247857
Твои выводы на чем основаны? Они на сколько корректны, так, примерно можешь почувствовать?
Аноним 13/01/16 Срд 18:50:31 #392 №247869 
>>247851
А какие по-твоему мнению законы современной физики нарушаются при объединении двух горизонтов событий? Деграссыч вон говорит, что матан сложный, там не просто слияние объектов в классическом понимании. Хотя его "хуе мое попадаем в прошлое" я не очень то доверяю.
Аноним 13/01/16 Срд 19:47:44 #393 №247879 
14527036646910.jpg
>>247869
Нигру и я слушаю вполуха; да и не очень ботанил, что там и как, при слиянии. Но слияние ничего не нарушает, просто процесс там посложнее, чем две звезды въебенились друг в друга.

Тут уже важнее, как два горизонта смыкаются, как меняется топология (хотя в результате всё равно получается сфера). Да, и слияния ЧД вроде как не должны порождать никакого ЭМ-излучения, вообще ничего, только гравитационные волны. Слились тихо и темно, только "рябь" по пространству. Хотя, наверное, столкновение аккреционных дисков такое порождает, что ёбаный пиздец.
Аноним 13/01/16 Срд 19:54:28 #394 №247880 
>>247879
А там гравитационное замедление времени вообще может дат ь им слиться с т.з. стороннего наблюдателя?
Аноним 13/01/16 Срд 20:01:28 #395 №247882 
>>247849
>>247855
Нихуя не изменится через 50 мегалет, солнце начнет пухнуть через 500 мегалет, умник.
Аноним 13/01/16 Срд 20:36:29 #396 №247895 
14527065900480.jpg
>>247880
>А там гравитационное замедление времени вообще может дат ь им слиться с т.з. стороннего наблюдателя?
Ну не начинай. Из треда в тред кочует парочка товаризчей, что доказывают невозможность пересечь горизонт ЧД на скорости, меньшей или равной световой. Теперь вот слияние.

Уже всё тысячу раз разжевано. Проблема здесь не в физике - в логике. На Землю телам ничто не мешает падать, если их скорость до этого была ниже второй космической. И это никак не возбуждает вышеупомянутых товаризчей; когда же дело доходит до ЧД - буйство просто.
Аноним 13/01/16 Срд 20:44:55 #397 №247905 
>>247895
Погодипогоди, по-моему ты меня неправильно понял (либо я вообще ничего не смыслю в теме).
Насколько я помню, из-за гравитационного замедления времени посторонний наблюдатель не увидит падения чего-либо под горизонт событий, для него это тело зависнет на горизонте. А что будет с двумя ЧД, что в этом случае увидит удаленный сторонний наблюдатель?
Аноним 13/01/16 Срд 21:03:08 #398 №247918 
14527081881680.png
>>247905
>А что будет с двумя ЧД, что в этом случае увидит удаленный сторонний наблюдатель?
Тут, думаю, только нигра поможет/сочинит очередное кинцо. Если ЧД не имеют аккреционных дисков (ага, конь в вакууме), то ничего не увидит. Только "волны" гравитации прокатятся по окрестностям. Специально же написал
>Да, и слияния ЧД вроде как не должны порождать никакого ЭМ-излучения
Так что "увидеть" ничего не получится.
Аноним 13/01/16 Срд 21:40:24 #399 №247928 
Вот кстати, насчет бесконечного падения. А если черная дыра испарится, или наоборот разрастется, как наблюдатель будет видеть бесконечное падение?
Аноним 13/01/16 Срд 21:54:01 #400 №247932 
>>247928
Скорее всего картинка падающего чего то будет медленно тускнеть и потом совсем пропадет.
Я тот анон, что спрашивал про разрушение чд.
Аноним 13/01/16 Срд 22:04:37 #401 №247935 
>>247626
как быть, когда кончатся углеводороды? ведь они - не только топливо и эфемерная "энергия", но и вполне осязаемый пластик, шмотки и прочее?
Аноним 13/01/16 Срд 22:05:18 #402 №247936 
Всем известно, что современные представления о чд говорят нам о наличии сингулярности внутри. То есть мы имеем бесконечную плотность, температуру и дальше по списку. И эти параметры ведь не должны различаться от размера чд? Плюс есть вопрос - а не превышается ли скорость света при непосредственном сближении горизонтов событий? Если нет, то она равна с. Я правильно все понимаю?
И совсем легкий вопрос - остается ли что либо от поглощенной чд в плане материи или еще чего либо даже на совсем короткий промежуток времени?
Аноним 13/01/16 Срд 22:22:22 #403 №247941 
14527129427710.jpg
>>247935
>как быть, когда кончатся углеводороды? ведь они - не только топливо и эфемерная "энергия", но и вполне осязаемый пластик, шмотки и прочее?
Вот ведь проблема! Когда-то не было земледелия и скотоводства, да и человеков было тыщ 70, и ничего - выжили. Теперь - 7 миллиардов рыл, технологии, ёбана мать, всё остальное. Пропадем без бензина, "Mad Max" устроим.
Аноним 13/01/16 Срд 22:26:16 #404 №247943 
>>247936
>мы имеем бесконечную
Кто "мы"? Да, и что такое нуль дзета-функции выучи, спрошу.

>И совсем легкий вопрос - остается ли что либо от поглощенной чд в плане материи или еще чего либо даже на совсем короткий промежуток времени?
На этот вопрос тебе уже ответил какой-то мимокрокодил - >>247856
Аноним 13/01/16 Срд 22:29:01 #405 №247945 
>>247935
Как это "кончатся"? Для того, чтобы их не стало, надо не только высосать недра насухо, но и выкинуть все пластики и прочие продукты (в т.ч. и углекислый газ после сжигания) в прожорливое чрево какого-нибудь Юпитера.
Тогда, очевидно, будем хуи кусать и локти сосать, и думать, как из Юпитера обратно карбон доставать.
Но углеводороды и их продукты никуда не деваются, так же валяются в атмосфере, гидросфере и литосфере, бесцеремонно выброшенные на ТБО и сожжённые в ДВС.
Деревья и фитопланктоны соберают все углекислые газы, которые видят. На два мешка целый день уходит. Прочие пластики разлагаются и в земле-матушке уходят на переработку обратно в нефть-батюшку.
Ну или более сознательные граждане срезают пару миллионов лет и перерабатывают пластики-шмотки во вторсырье.
Что же до топлив и всяких пластмасс - вовсю цветут и пахнут биодизеля, биопластики и прочий эрзац-суррогат, замещающий оригинал.

>>247936
>Всем известно, что современные представления о чд говорят нам о наличии сингулярности внутри.
В моих представлениях сингулярности внутри ЧД нет. Но не будем об этом.
>То есть мы имеем бесконечную плотность, температуру и дальше по списку.
Это математическая абстракция. Если вооружиться некоторыми представлениями, то вполне можно утверждать, что размеры той самой сингулярности ограничены кубической планковской длиной. С учетом того, что масса конечна, ни о какой бесконечной плотности речи и не идет.
>И эти параметры ведь не должны различаться от размера чд? Плюс есть вопрос - а не превышается ли скорость света при непосредственном сближении горизонтов событий? Если нет, то она равна с. Я правильно все понимаю?
С чего бы константе меняться?

>И совсем легкий вопрос - остается ли что либо от поглощенной чд в плане материи или еще чего либо даже на совсем короткий промежуток времени?
Хуй знает. Мы можем говорить о принципах работы ЧД с тем же успехом, что и рассуждать о красочных миниатюрных лошадках, вооруженных магией.
Есть хитрый матаппарат, которым полтора инвалида вроде Хокинга владеют, есть всякие Грины и те же Хокинги, которые его разжевывают и пережевывают каждый год по-новому. Можно ознакомиться, если интересно.
Если вкратце - ебучий матан, мозгоразрывающий пространственно-временной континуум, который не понять, не порвав шаблоны и спекуляции-спекуляции-дружба-магия.
Аноним 13/01/16 Срд 22:51:05 #406 №247951 
>>247945
давай чтоли постройку хабитата обсуждать, ты вроде норм
Аноним 13/01/16 Срд 23:10:10 #407 №247959 
>>247951
А что с хабитатом?
Денег нет, проектов нет.
Пока хотя бы петлю Лофстрома не построят, не засылать нам больше десятка ну сотни человеков за пределы планеты.
Аноним 13/01/16 Срд 23:37:17 #408 №247967 
14527174378410.jpg
>>247959
>не засылать нам больше десятка ну сотни человеков за пределы планеты
Пока не появятся цели/задачи. Пока дядьки с толстым кошельком не дадут денег. Пока не будет целесообразности. Все тут знают, что военный бюджет США/РФ/Китая раз в сто больше бюджета космических программ.

Да, тупая нажива путем спекуляции на биржах есть. Есть ненавидимые вами "айфоны" - общество потреблябства вместо общества технофашизма. Только вот об экономике знаете вы столько же, сколько эскимос об отоплении.

Да, всем бы нам хотелось картинок с подледных океанов Энцелада посмотреть, но правила есть правила. Хочешь таких картинок - пили АМС и бур с аппаратами сам. Остальных "и здесь неплохо кормят".
Аноним 14/01/16 Чтв 00:18:04 #409 №247972 
>>247967
>Все тут знают, что военный бюджет США/РФ/Китая раз в сто больше бюджета космических программ
Справедливости ради те же траты без профитов как и космос. Например на программу F-22 было потрачено четыре годовых бюджета НАСА, а именно $74 млрд (данные с вики), произведено около 200 самолётиков. Польза? Пару десятков раз сбрасывали чугуний на бабахов в Сирии. С чем справился бы любой бомбардировщик времён второй мировой.

Так что у космоса есть шанс безо всякой целесообразности. Просто потому что интересно и потому что надо же чем-то заниматься высокоразвитому существу.
Аноним 14/01/16 Чтв 00:41:51 #410 №247977 
>>247972
>Справедливости ради
Да ведь не очень справедливо. Я не военачер, вообще далек от военной техники, но про американские самолеты "F-..." - много слышал.

Тем не менее - говорил я об экономике. И деньги, в 10 раз большие, чем на космос, для spc-дрочеров - трудно перевариваемый факт. Потому и написал о невозможности технофашизма.
Аноним 14/01/16 Чтв 01:37:14 #411 №247984 
>>247959
деньги можно достать, но куда их потом спихивать

>>247967
>Пока дядьки с толстым кошельком не дадут денег.
не стоит ждать чужих милостей

Давайте слегка абстрагируемся от денег, если нет идеи, конецепции или эти идеи и концепции не реализуемы требуют превышения скорости света к примеру, то деньги в этом случае не помогут.

Нет, конечно, если бы у меня были деньги, я бы просто нанял специалистов которые бы прорабатывали тех детали.

Вообщем хотелось бы просто обсудить сам сабж, и варианты его постройки, с тем что есть на данный момент.
Аноним 14/01/16 Чтв 01:51:22 #412 №247985 
>>247984
Да перестань. США вести войны в другой части света - равнозначно пилить космопрограмму. Но они ведут войны, а РН забыты. Хотя, ты говорил об "абстрагироваться от всего, в том числе денег". И мы снова приходим к технофашизму: космос ради космоса.
Аноним 14/01/16 Чтв 02:18:14 #413 №247988 
>>247985
как буратино чесслово, но ок, я понимаю, есть два типа людей, одним нужны деньги, а другие сидят на трубах.
ок, предположим есть организация, с оборотом в 10 миллиардов баксов, и годовой прибылью в 3 миллиарда, 1ккк в год на протяжении следующих 15 лет они готовы потратить на проект.
Следуя духу насаидеии на халяву, они объявили конкурс с банком в 1 бакс и золотым билетом, за лучший план концепцию по постройке орбитальной станции на 1кк людей.
Ваши предложения. В рамках имевших место или текущих технологий.

Обращаю внимание, это не план станции как таковой. Это набросок пути, который может привести к ее постройке.
Это этапы и области вложения средств. Некоторая блоксхема, которая в конечном итоге приведет к реализации, постройке станции.
Аноним 14/01/16 Чтв 02:24:46 #414 №247989 
>>247984
>Нет, конечно, если бы у меня были деньги, я бы просто нанял специалистов которые бы прорабатывали тех детали.
Предполагаешь, что деньгиимущие таких спецов не наняли?
Аноним 14/01/16 Чтв 02:32:29 #415 №247990 
>>247988
>Это этапы и области вложения средств. Некоторая блоксхема, которая в конечном итоге приведет к реализации, постройке станции.
В этой блок-схеме отсутствует один блок: прибыль. Дядьки с большими кошельками имеют таки кошельки ровно по одной причине: они умеют считать прибыль.
Аноним 14/01/16 Чтв 03:04:19 #416 №247993 
>>247567
Не пизди, физика в 7ом классе появляется.
Аноним 14/01/16 Чтв 03:07:57 #417 №247994 
>>247623
>постоянная
>растет
Ох уж эти астрофизики.
Аноним 14/01/16 Чтв 03:25:13 #418 №247995 
14527311140420.gif
>>247994
Да, "ох уж эти астрофизики". Постоянная Хаббла лишь условно называется так. Она постоянная в каждой точке Метагалактики, в любой момент времени. Но на непостоянна в эпохах: считается, что она начала расти 5 млрд. лет назад, а в Инфляционную Эпоху она вообще была ебись конем.
Поясните в чем автор статьи не прав Аноним 14/01/16 Чтв 03:40:14 #419 №247996 
14527320142990.png
Есть версия, что про то, что прежде чем лететь на луну, нужно понять что она такое.

«С Луной связан целый ряд совершенно вопиющих фактов несоответствий с устоявшимися воззрениями классической физики, которые в литературе и Интернете стыдливо называются «лунными аномалиями».

Самая очевидная аномалия – точнейшее совпадение периода обращения Луны вокруг Земли и вокруг своей оси, из-за чего она всегда обращена к Земле одной стороной. Существует множество причин, чтобы эти периоды всё больше рассинхронизировались на каждом витке Луны вокруг Земли. Например, никто не станет утверждать, что Земля и Луна являются двумя идеальными шарами с равномерным распределением массы внутри.

С точки зрения официальной физики совершенно очевидно, что на движение Луны существенное влияние должны оказывать не только взаимное расположение Земли, Луны и Солнца, но даже пролёты Марса и Венеры в периоды максимального сближения их орбит с земной. Опыт космических полётов на околоземной орбите показывает, что достичь стабилизации по типу лунной можно только в том случае, если постоянно подруливать микродвигателями ориентации. Но чем и как подруливает Луна?

Странная кривая, которую Луна описывает над нашими головами, согласуется всего лишь с длинным списком статистических параметров, изложенных в соответствующих таблицах. Эти данные собраны на основе многолетних наблюдений, но отнюдь не на базе каких-либо расчётов. Именно благодаря этим данным можно предсказать те или иные события с большой точностью, например, солнечные или лунные затмения, максимальное приближение или удаление Луны относительно Земли и т.д.

Так вот, именно на этой странной траектории Луна ухитряется всё время быть развернутой к Земле только одной стороной! Конечно же, это далеко не всё.

Оказывается, Земля двигается по орбите вокруг Солнца отнюдь не с равномерной скоростью, как хотелось бы официальной физике, а делает небольшие притормаживания и рывки вперёд по направлению своего движения, которые синхронизированы с соответствующим положением Луны. Официальная физика не только не берётся описать или объяснить эти процессы – она о них просто умалчивает! Такой полумесячный цикл рывков земного шара отлично коррелирует со статистическими пиками землетрясений, но где и когда вы об этом слышали?»

http://ss69100.livejournal.com/2443166.html
Аноним 14/01/16 Чтв 03:49:31 #420 №247998 
>>247996
> Существует множество причин, чтобы эти периоды всё больше рассинхронизировались на каждом витке Луны вокруг Земли.
g "либрация"
> Опыт космических полётов на околоземной орбите показывает, что достичь стабилизации по типу лунной можно только в том случае, если постоянно подруливать микродвигателями ориентации.
Опыт полетов и физика подсказывает, что достаточно сделать аппарат в виде гантели подлиннее с блинами разной массы, и градиент гравитации сам его развернет как нужно.
> Оказывается, Земля двигается по орбите вокруг Солнца отнюдь не с равномерной скоростью, как хотелось бы официальной физике, а делает небольшие притормаживания и рывки вперёд по направлению своего движения, которые синхронизированы с соответствующим положением Луны. Официальная физика не только не берётся описать или объяснить эти процессы – она о них просто умалчивает!
Как страшно жить, физики всего мира утаивают то, что Луна и Земля вертятся вокруг общего центра тяжести, который не совпадает с центром Земли.

> Все естественные науки, в первую очередь физика, химия, геология и астрономия, подверглись таким процедурам догматизации, выхолащивания сути и доведения до абсурда, что теперь любой корректно поставленный эксперимент всегда показывает расхождение с официальными теориями.
Уууу, хорошее начало, там, небось, дальше про образованцев и автора в белом пальто? Вот и ссылки про уничтожение торсионных исследований вижу.

В общем, автор статьи не прав в том, что он кефирщик если не буквально (лень вчитываться), то по духу.
Аноним 14/01/16 Чтв 03:50:42 #421 №247999 
>>247996
Такой-то тонкий жирок. Анончег, покорми обязательно эту няшу.
Я то только вот это знаю https://en.wikipedia.org/wiki/Tidal_locking
но может еще чего.
Аноним 14/01/16 Чтв 04:11:45 #422 №248000 
14527339051230.jpg
>>247999
Да какой это тонкий жирок? Тут впору банить, насовсем.
Аноним 14/01/16 Чтв 04:41:44 #423 №248002 
>>248000
протестую
Аноним 14/01/16 Чтв 05:34:41 #424 №248004 
>>247993
Это в каких-то задрипанных шарагах, наверное. У меня с третьего класса физика была. Второй иностранный - с 5 класса, информатика - с первого.

>>247996
Бляяя.
АААААПЧХИ!
Простите, у меня аллергия на ахинею.

>>248000
>Тут впору банить, насовсем.
Двачую.
Когда какой-то уёбок пишет "Ученые скрывают", не удосужившись хотя бы википедию почитать, где вот они, все основные факты и пруфы, вышеупомянутые либрации и приливное замыкание, которое наблюдается и у Фобоса, Деймоса, крупных спутников Юпитера и Сатурна и Платона-Харона.
Аноним 14/01/16 Чтв 06:25:54 #425 №248008 
>>248004
Сыш, илюминат, шел бы ты отсюда.
Аноним 14/01/16 Чтв 11:53:08 #426 №248030 
>>247989
Тебе какой то профит с этого? Или к примеру есть интересная инфа в открытом доступе?
Если есть скидывай линку.

>>247990
>Дядьки с большими кошельками имеют таки кошельки ровно по одной причине: они умеют считать прибыль.
>по одной причине
Это не так. Считать учат в школе.

Аноним 14/01/16 Чтв 12:33:30 #427 №248037 
>>248004
я отмечу один момент
на такие шаблонно тупые вопросы должны быть достаточно простые ответы
и возможно имеет смысл их напоминать периодически
Вообще уровень детских почемучек, и может просто пригодится
Аноним 14/01/16 Чтв 12:37:39 #428 №248039 
>>247972
>Пару десятков раз сбрасывали чугуний на бабахов в Сирии.
>F-22
Ебанат? Ебанат.
Аноним 14/01/16 Чтв 12:50:40 #429 №248046 
>>248037
Но это не тупые вопросы. "Тупой" вопрос - это "А че луна всегда одной стороной смотрит, она не крутится что ли?".
А это - показное напускное невежество, "Всем известно, что...", "Ученые доказали, что...", "Нет никаких свидетельств, что..." - это симптом тупорылой мрази со съеденными каналом рентв мозгами, и место этой гнили с упомянутым каналом - в биореакторе.
Аноним 14/01/16 Чтв 12:57:28 #430 №248052 
14527654481860.png
14527654481881.png
Взлетит?
Аноним 14/01/16 Чтв 13:40:59 #431 №248067 
>>248004
>У меня с третьего класса физика была. Второй иностранный - с 5 класса, информатика - с первого.
И кем ты стал по жизни?
sageАноним 14/01/16 Чтв 13:44:32 #432 №248068 
>>248067
>кем ты стал по жизни
>хвастается своей школьной программой на дваче
Щас он тебе расскажет что такое успех.
Аноним 14/01/16 Чтв 13:47:13 #433 №248070 
>>248067
>сижу на дваче
А то ты не догадываешься.

>>248052
Тяга у первой ступени - 680 тонн, масса первой ступени 450 тонн, плюс 40 тонн ориджина - 490 тонн. Да, взлетит. Далеко ли - другой вопрос. Если тупо дельты просуммировать - то на орбиту, но сдается мне, тут не все так просто. Да и банка обратно долетит в виде расплава, т.к. на сход с орбиты не расчитана.
Аноним 14/01/16 Чтв 13:52:57 #434 №248076 
Сегодня 16 лет Гюйгенсу. Почему-то вообще не помню ничего про него в новостях в то время, как-то не форсилось.
Где найти архив всех фотографий, которые он передал?
Аноним 14/01/16 Чтв 14:14:00 #435 №248079 
>>248070
>А то ты не догадываешься.
Не, я без попыток унизить. Просто реально интересно, как такая очевидно перегруженная программа влияет на людей? Сириусли, в моей шкалке физика стандартно началась с 7-го класса. И все бы ничего, но к 9-му классу у нас появился гуманитарный уклон, потому что типа назывались гимнасией, а учебный план стандартный. Физику укоротили до 2 часов в неделю и наш препод, который любил поматюгаться, счел, что двух часов недостаточно для полноценного закрепления программы физики старших классов, а потому обучать нас нет смысла и на уроках он нам травил кул-стори из своей жизни. Контрольные мы делали по шаблону задач из двух действий, которые он предварительно разбирал на уроке, меняя только цифры для варианта. Особенно одаренным это не мешало, тем не менее, писать эти контрольные на тройки.
Так вот как-то уже в одиннадцатом классе он мне предложил посещать дополнительные занятия по субботам, которые он вел для тех, кто все же собирался писать ЕГЭ по физике. Я тогда отмахнулся, потому что угорал по географии.
Вооот, потом в вузике, а я поступил на геодезию, не то, чтобы сильно связанную дисциплину, но у нас пошла очень такая солидная физика. Ну как солидная, кто-то и в школе это изучает, но это была механика во всей полноте. Мне было оче непросто, я нихуя не понимал, и удовлы свои получил скорее из жалости.
Однако потом, года через три уже после окончания курса, меня как-то пробило на увлечение. И я сам, своим умом, внезапно понял механику и статику, проникся и прочувствовал. Очень вовремя, надо сказать, что помогло мне пройти курсы гравиметрии и небесной механики на отлично. Я настолько стал чувствовать "пульс науки", что захотелось снова пройти через первый курс физики, потому что те задачки на баллистические траектории стали кристально прозрачными в своей сути, а теорема Штейнера показалась очевидной и простой. Выводы формул для тел разных форм из нее приобрели в моем взгляды черты элегантности. Но долой романтику, справедливости ради скажу, что воспоминания о втором курсе таки до сих пор приводит к жопным болям. Электродинамика мне на далась, от мыслей о законе Био-Савара-Лапласа по прежнему передергивает. Волновая оптика туда же. Да, матан был потяжелее, но причина все же в том, что душа так и не легла.
К чему я все это? Да к тому, что в уровне понимания механики наверняка дам фору 90% твоих одноклассников, и в упор не представляю, что можно запихать из физики в школьную программу на 9-ть лет.
Аноним 14/01/16 Чтв 14:25:09 #436 №248083 
>>245254 (OP)
Аноны, вот тут все ловят багет от того что наша вселенная расширяется, расстояние между галактиками все больше и больше, и чем оно больше, тем быстрее отдаляемся. Но в нашей галактике остается 200 млрд звезд, охуенная черная дыра в центре, которая всех держит при себе, и все заебись. Зачем нам вся остальная видимая вселенная? Нашей цивилизации, если она когда нибудь действительно станет разумной, чтобы совершать межзвездные перелеты, вполне ее хватит. Все заебись и стабильно у нас. Правда пизда Андромеда "скоро" хуйню тут натворит. Тут и возможны появления квазаров, мало ли прилетит вещества в массивную черную дыру. И тд.
Аноним 14/01/16 Чтв 14:25:55 #437 №248084 
>>248083
а так то все ок. То что нахуя нам остальная вселенная?
Аноним 14/01/16 Чтв 14:35:48 #438 №248085 
>>248084
Никто и не собирается гонять в хаблдипфилд за хлебом. Это космология, которая помогает лучше понять физику. Увидел объект, натянул на него теорию — подтвердилась, супер. Или нет, тоже результат. Или наоборот, не натянул теорию, а придумал новую и айда в ближайшие окрестности искать подтверждение.
Аноним 14/01/16 Чтв 15:40:43 #439 №248093 
>>248083
>вот тут все ловят багет
Кто? Не замечал таких багетов. Просто по одной из моделей через 22 млрд лет придет большой разрыв и всю материю распидорасит на атомарном уровне в любой точке вселенной из-за экспоненциального роста постоянной Хаббла.
Кого-то печалит, что никаких способов преодолеть большое сжатие или большой разрыв постулаты современной физики не позволяют. Но у других есть и оптимистично-дарвинистский взгляд. Типа если за 20 млрд лет люди не научатся прыгать в гиперпространство и 26-ые измерения, то туда им и дорога и нахуй они такие не нужны.
Аноним 14/01/16 Чтв 15:51:09 #440 №248094 
>>248079
> Просто реально интересно, как такая очевидно перегруженная программа влияет на людей?
Да никак, по-моему. Йоба-гимназия же, тупарей не держали, т.е. было как в обычной школе, просто побольше всего.
В итоге первый курс я закончил на пятерки просто по инерции, т.к. все-все-все уже было пройдено в 10-11 классах, как-то: дифуры, погромизм, экономика и общие знания вроде всяких правоведений.
Т.к. я прошел первый курс по инерции, то нехуево расслабился и пошел во все тяжкие из-за чего и вылетел нахер потом.
>>248079
>К чему я все это? Да к тому, что в уровне понимания механики наверняка дам фору 90% твоих одноклассников, и в упор не представляю, что можно запихать из физики в школьную программу на 9-ть лет.
Наверняка. Практически никто вне сферы своей деятельности или интересов не поддерживает знания, а многие и в школе их усваивают постольку-поскольку.
А я вот астрономией увлекся и читаю всякое, потому тут заседаю и отвечаю практически на любые вопросы.

>>248083
>Все заебись и стабильно у нас.
Эм-м, нет. Приятель, расширается ВСЯ вселенная, притом с ускорением. Сегодня галактике похуй, а завтра (через какие-нибудь двадцать гигалет) и ей жопа.
Видишь ли в чем дело, увеличивающаяся вселенная удаляет все объекты друг от друга, и в какой-то прекрасный момент скорость удаления превышает скорость света, и наблюдать и взаимодействовать с объектом мы можем примерно никак. Пока это касается ебически удаленных галактик, а потом будут удаляться и ближние галактики, не способные компенсировать расширение вселенной гравитацией, а затем и звезды в нашей галактике, а затем и ближайшие звезды, потом и солнце потухнет, и за короткий промежуток времени, разорвет и тебя, и молекулы, и атомы.
Вот поэтому и ловят багет.
Аноним 14/01/16 Чтв 15:53:57 #441 №248096 
>>248085
тру

давай хабитаты обсуждать
Аноним 14/01/16 Чтв 15:54:09 #442 №248097 
14527760497630.jpg
>>248093
>экспоненциального роста постоянной
Аноним 14/01/16 Чтв 15:55:48 #443 №248099 
14527761486680.jpg
>>248094
>скорость удаления превышает скорость света
Но... Ведь... ОТО... Эйнштейн...
Аноним 14/01/16 Чтв 16:01:01 #444 №248104 
>>248099
СТО, вообще-то.
И нет, оно неприменимо. Здесь речь не идёт о перемещении объектов, а о расширении структуры самого пространства, в котором эти объекты расположены. Это всё равно, что наблюдать перемещение намертво вбитой в лед полярной станции на льдине. Станция же не должна двигаться, и она не движется как бы, но лед, на которой она - движется.
Ну или дозвуковой самолет в джетстриме относительно земли летящий на сверхзвуке может быть примером.
Аноним 14/01/16 Чтв 16:05:35 #445 №248109 
>>248104
эх такой добрый , а потом двач испортит
Аноним 14/01/16 Чтв 16:05:56 #446 №248110 
>>248104
>расширении структуры самого пространства
Если оно может расширяться, значит может и сужаться.
Значит, его можно сузить и поддерживать в таком состоянии, если есть энергия на это.
Аноним 14/01/16 Чтв 16:08:59 #447 №248111 
>>248110
Отлично.
Садись за матан, к 70 получишь нобелевку, если придумаешь, как управлять структурой пространства. Потому, что пока что идей нет.

>>248109
Не понел.
Аноним 14/01/16 Чтв 16:21:46 #448 №248113 
>>248111
> Садись за матан, к 70 получишь нобелевку, если придумаешь, как управлять структурой пространства. Потому, что пока что идей нет.
Вон, Уайт пытается электрическим полем управлять, так что идеи есть.
Аноним 14/01/16 Чтв 16:28:06 #449 №248115 
>>248113
Пока нихуя не вышло же, хотя столько лет уже прошло.
И вообще, как магнитные поля с гравитацией связаны?
Аноним 14/01/16 Чтв 16:29:50 #450 №248117 
>>248097
Опять начинаешь, ебучий шакал!?
Аноним 14/01/16 Чтв 16:46:16 #451 №248120 
Спейсач, объясни мне, ссаному гуманитарию, как квазары испускают свой луч света? Ведь фотоны, чтобы вылететь из-за горизонта событий, должны развить скорость больше скорости света, а это, как известно, невозможно. В чем подвох?
Увлекся космологией недавно, так что я ещё зелёный.
Аноним 14/01/16 Чтв 16:48:03 #452 №248121 
>>248120
Так они и не попадают за горизонт событий. И светятся не сами фотоны, а разогретая плазма на релятивистских скоростях, т.н. джеты. Большая часть аккреционного диска падает в черную дыру, а части вылетают с полюсов.
Аноним 14/01/16 Чтв 17:25:56 #453 №248126 
Какой шанс на открытие вапр-пихла или алиены, вторжение, рабство?
Аноним 14/01/16 Чтв 17:30:46 #454 №248129 
>>248126
Ненулевой.
Аноним 14/01/16 Чтв 17:40:20 #455 №248132 
>>248083
Ты охуел, а на что я в телескоп тогда смотреть буду?
Аноним 14/01/16 Чтв 18:41:00 #456 №248147 
>>248132
На мать свою, откроешь на ней лицо, рот там и детали всякие, карту составишь. Потом статью напишешь в научный журнал, а мы почитаем.
Аноним 14/01/16 Чтв 18:49:50 #457 №248148 
>>248094
>Эм-м, нет. Приятель, расширается ВСЯ вселенная, притом с ускорением. Сегодня галактике похуй, а завтра (через какие-нибудь двадцать гигалет) и ей жопа.
>Видишь ли в чем дело, увеличивающаяся вселенная удаляет все объекты друг от друга, и в какой-то прекрасный момент скорость удаления превышает скорость света, и наблюдать и взаимодействовать с объектом мы можем примерно никак. Пока это касается ебически удаленных галактик, а потом будут удаляться и ближние галактики, не способные компенсировать расширение вселенной гравитацией, а затем и звезды в нашей галактике, а затем и ближайшие звезды, потом и солнце потухнет, и за короткий промежуток времени, разорвет и тебя, и молекулы, и атомы.
>Вот поэтому и ловят багет.
Схерали?? То что галактики разлетятся оно понятно, а с чего бы это должно разорвать галактику? Черная дыра будет на месте некуда не испарится. Гравитация будет держать, все при нас. Ну выгорят большая часть звезд, новые образовываться не будут, на небе не станет звезд. Но я не вижу причин по которым в течении предстоящих 200 млрд лет вдруг все развалится на частицы.
Аноним 14/01/16 Чтв 18:53:20 #458 №248152 
>>248148
>Но я не вижу причин
Причина называется w<−1. Никто громких утверждений не делает, но ты бы хоть погуглил "Большой разрыв" штоле.
Аноним 14/01/16 Чтв 19:15:46 #459 №248161 
>>248148
>Схерали?? То что галактики разлетятся оно понятно, а с чего бы это должно разорвать галактику? Черная дыра будет на месте некуда не испарится. Гравитация будет держать, все при нас.
Между СМЧД в центре галактики и удаленной звездой сейчас образуется 1.3 километра пространства каждую секунду.
Если считать по постоянной Хаббла на текущий момент: 67.15 ± 1.2 (km/s)/Mpc 1 и расстоянием до звезды в 20 kpc от центра.
Можно смело предположить, что при расстоянии в 610^20 метров (20 килопарсек) какие-то 1.310^3 метров пренебрежимо малы.
В случае, если гипотеза о фантомной энергии и ускоряющемся расширении Вселенной верна, то со временем будет образовываться гораздо больше пространства, и в какой-то момент (предположительно за 60 мегалет до пиздеца) между звездой и СМЧД будет образовываться его столько, что гравитация уже не сможет компенсировать это.
Вкратце здесь это и написано:
https://en.wikipedia.org/wiki/Big_Rip

1https://en.wikipedia.org/wiki/Hubble%27s_law
Аноним 14/01/16 Чтв 20:26:14 #460 №248180 
>>248111
как же серая энергия, черное антивещество ? Алькубьерре это наше все FTL фанатов.

>Не понел.
проехали
Аноним 14/01/16 Чтв 20:27:42 #461 №248181 
>>248180
Говори более связно, пожалуйста.
Темную энергию знаю, антивещество тоже, а что ты написал распарсить не могу.
Аноним 14/01/16 Чтв 20:31:55 #462 №248185 
>>248181
я про это https://en.wikipedia.org/wiki/Alcubierre_drive
и отрицательную или что там энергию, потребную для создания этой колесницы диавола
Ну сорь, не могу без сарказма на эту тему
Аноним 14/01/16 Чтв 21:09:56 #463 №248205 
Какие исследования или измерения могут продвинуть существующую космологию дальше? Кроме темной материи\энергии
Аноним 14/01/16 Чтв 21:11:06 #464 №248206 
Способна ли чд поглощать антиматерию? Происходит ли на этом этапе аннигиляция?
sageАноним 14/01/16 Чтв 21:14:25 #465 №248207 
>>248206
Антиматерия обладает массой, так что если ЧД сожрет антиматерию, то она тупо увеличит свою массу на массу сожратой антиматерии. Что там будет внутри ЧД - уже не важно, даже если там произойдет аннигиляция, то энергия останется внутри ЧД, закон эквивалентности массы и энергии говорит о том, что совершенно неважно, случится аннигиляция, или нет.
Аноним 14/01/16 Чтв 21:47:08 #466 №248212 
>>248205
Поиск гравитационных волн.

>>248206
Да. Мне кажется, что можно считать, что поглощается не какая-то масса с характеристиками антивещества, звезды или мимокрокодила, а энергия, соответствующая поглощаемому веществу.

Кстати, поясните за теорему "без волос". Почему информация не может теряться, кто сказал такое? Было вещество - перестало существовать, стало энергией и хуй с ней.
Аноним 14/01/16 Чтв 23:31:40 #467 №248257 
14528035009950.jpg
>>248212
>Почему информация не может теряться, кто сказал такое?
Изменяются суммарные характеристики Метагалактики, которые, в общем-то, должны оставаться постоянными. Например, энтропия, напрямую завязанная на массу. А законы сохранения такого позволять не должны.

Это, кстати, открытая проблема космологии. Потеря информации происходит при излучении Хокинга - частном случае эффекта Унру. Да и сами ЧД не добавляют радости ученым.
Аноним 14/01/16 Чтв 23:50:58 #468 №248260 
>>248257
>Кстати, поясните за теорему "без волос"
И теорема эта слабо связана с информацией. Проще представить, что из пространства "выкусили" кусок, и всё. Поэтому никаких внешних характеристик и нет: это не объект в классическом понимании, это не сверхплотная звезда. Просто нет куска пространства, "дыра".
Аноним 14/01/16 Чтв 23:52:39 #469 №248263 
>>248207
Н-но ведь в результате аннигиляции масса ЧД уменьшиться? И даже может стать ниже критической?
Аноним 15/01/16 Птн 00:06:07 #470 №248269 
>>248263
>в результате аннигиляции
Аннигиляции с чем?
Аноним 15/01/16 Птн 00:09:14 #471 №248272 
>>248263
Что что?
sageАноним 15/01/16 Птн 01:06:49 #472 №248282 
>>248263
>в результате аннигиляции масса ЧД уменьшиться?
Перечитывай тот ответ снова и снова, до полного просветления. >>248207
Аноним 15/01/16 Птн 02:57:09 #473 №248293 
>>248282
Не понял что такое антиматерия. Это ядро со знаком минус и позитроны вокруг него или материя со знаком минус масса?
Аноним 15/01/16 Птн 03:16:57 #474 №248300 
Почему на Луну уже много лет не никто не хочет летать? Что там такое страшное обнаружили?
Аноним 15/01/16 Птн 03:30:46 #475 №248303 
>>248300
Почему ты в Антарктиду не летаешь?
Аноним 15/01/16 Птн 03:33:50 #476 №248304 
>>248303
Денег нет
Аноним 15/01/16 Птн 03:43:03 #477 №248309 
>>248304
И шмель длинный?
Аноним 15/01/16 Птн 10:35:44 #478 №248353 
>>248282
Ой, не пизди, энергия, в которую превратится масса после аннигиляции, не создает силы гравитационного притяжения. Так что в результате аннигиляции масса и сила притяжения ЧД уменьшится. Если масса втянутой антиматерии будет достаточно велика, то ЧД может перестать быть ЧД. Другое дело, что надо считать сколько времени займет эта аннигиляция. Возможно внутри ЧД она займет бесконечное время по часам внешнего наблюдателя и тогда похуй на все.
Аноним 15/01/16 Птн 10:41:00 #479 №248355 
>>248353
>энергия, в которую превратится масса после аннигиляции, не создает силы гравитационного притяжения
Больше охуительных историй, хороших и разных.
Аноним 15/01/16 Птн 11:07:21 #480 №248356 
>>248304
Вот и ответил. И не сильно хочется наверное?
Аноним 15/01/16 Птн 11:26:45 #481 №248360 
>>248356
Но на Луну же летали, а я а Антарктиду нет. Значит находили деньги.
Аноним 15/01/16 Птн 11:34:25 #482 №248361 
>>248353
>энергия, в которую превратится масса после аннигиляции, не создает силы гравитационного притяжения.
Альберт вращается с релятивисткой угловой скоростью из-за твоего наплевательского отношения к эквивалентности массы и энергии.
Аноним 15/01/16 Птн 11:45:07 #483 №248363 
14528475071230.jpg
Если телепортироваться на 100 световых лет, то то место будет выглядеть так же, как с земли в телескоп, сейчас или как на земле через 100 лет?
Аноним 15/01/16 Птн 12:04:00 #484 №248367 
>>248363
Зависит от способа телепортации
Аноним 15/01/16 Птн 12:29:11 #485 №248374 
>>248363
100 лет пройдет, пока тебя на том месте заметят с Земли.
Аноним 15/01/16 Птн 12:33:22 #486 №248375 
>>248363
Если хакнешь физику и моментально для всех-всех-всех переместишься, то так, как через 100 лет. Но пока физика не хакается и ты можешь туда теоретически переместиться за 1*10^-100 секунд, ебаша на околосветовой, например, но для земли и того места уже 100 лет пройдет.
Сверхсветовые перемещения нарушают принцип причинности, говорят, и я даже как-то раз понял, почему, там было хитрое объяснение с перекладываниями световых конусов, но я позабыл. Потому, что даже самое простое объяснение было сложноватым, чтобы с ходу раз и навсегда запомнить, а уж тем более, запомнить так, чтобы рассказать любому дауну.
Аноним 15/01/16 Птн 12:35:24 #487 №248376 
>>248363
Ты сначала подумай что будет если ты посмотришь оттуда на себя настоящего.
Аноним 15/01/16 Птн 12:44:13 #488 №248377 
>>248212
> Поиск гравитационных волн.
Так нашли же.
Аноним 15/01/16 Птн 12:50:57 #489 №248380 
>>248377
Где это нашли?
Ссылку на научную публикацию, пожалуйста.
Статья в стиле "Ученый изнасиловал журналиста" про BICEP таковой не является, если что. Главтетенька там говорит, что еще нихуя не понятно и надо перепроверять.
Аноним 15/01/16 Птн 13:05:25 #490 №248381 
>>248380
http://www.popsci.com/physicist-tweets-rumor-that-gravitational-waves-may-have-finally-been-detected
Аноним 15/01/16 Птн 13:13:32 #491 №248384 
14528528120900.jpg
14528528120911.gif
14528528120922.jpg
14528528120993.gif
>>248381
>tweets
>rumor
Лооооол.
Аноним 15/01/16 Птн 13:16:15 #492 №248385 
>>248384
Когда будут обнародованы официальные подтверждающие результаты, жду твоих извинений.
Аноним 15/01/16 Птн 13:25:10 #493 №248388 
>>248385
А за что мне извиняться?
Я априори не воспринимаю твиты, слухи и публикации в говносми за пруф. Если так делать, то можно в любую хуиту поверить: сверхсветовые нейтрино, школьник из уфы изнасиловал Гаусса изобрел гаусс-ган, нибиру набигает, Алла Пугачева приняла ислам.
Подобной хуиты по сто миллиардов постов каждую секунду строчат, и никак нельзя ее воспринимать всерьез.
Публикация же в научном peer-reviewed журнале - достаточно весомый аргумент верности теории. Вот когда подтвержденную публикацию притащишь - тогда и можно говорить.
Иначе это звучит словно "Тетя Йоба, уборщица из Дубны, сказала, что знакомый сына мужа ее внучатой племянницы, который летал во Францию, слышал в кабаке, что в ЦЕРНЕ зделоли черную дыру и не отпускают, когда подтвердят инфу - будешь извиняться", лол.
Аноним 15/01/16 Птн 13:30:43 #494 №248389 
14528538439510.jpg
>>248388
Ты что такой серьезный?
Аноним 15/01/16 Птн 13:37:20 #495 №248390 
>>248389
Ты что такой клоун?
Аноним 15/01/16 Птн 13:51:04 #496 №248395 
>>248390
Клоун! Меня называют клоуном! Вот так поворот. Выплывший из глубин подсознания поток абсолютного недоверия и полного безразличия к происходящему. Аллегория на существование, просто миф, пшик для вселенной, но какой глубокий. Единожды выскажи свое недоверие или веру, так тебя тут же освистают и закидают камнями, только дай повод! На расправы мы всегда были легки, скородумны. Но, кто по вашему задумывается о последствиях? Бесцельные и бездумные скопления самостей, просто порочащие сам факт существования разума. Я плюю вам в лицо, со всей своей широкой и щедрой руки, так, что вы в муках захлебнетесь, а на сдачу на том самом месте еще тысячу лет будут расти прекрасные сады. Я преклоняюсь перед вашим благоразумием и безумством, оно достигло апогея за какой-то миг, момент, долю секунды. Расцвело, раскинулось, словно чума, пожирая все больше людских душ с каждой минутой. Но это все блеф. Мы все мертвы с самого рождения. Сколько лет прошло, а запах гнили становится все приятней, ароматней, ближе к самой сути нашего существования. Я не хочу принимать в этом участия.
Аноним 15/01/16 Птн 13:56:06 #497 №248396 
>>248360
Нашли, потому что надо было дать пососать Иванам. После Аполло-11 задач у Луны не стало.
Аноним 15/01/16 Птн 16:05:38 #498 №248438 
>>248389
Джокер такой философский))
Аноним 15/01/16 Птн 19:29:50 #499 №248587 
Через полчаса Китай ракету запускает, где трансляцию смотреть, если будет?
Аноним 15/01/16 Птн 19:32:49 #500 №248592 
Я не знаю куда писать, в политач не хочу.
В общем, сейчас цена на нефть падает, даже в правительстве россии уже говорят что эра дорогой нефти заканчивается.
Я думаю это потому что уже научились получать более дешевую, экологическую и простую энергию. Те же ИТЕР.
Что думаете?
Аноним 15/01/16 Птн 19:33:18 #501 №248594 
>>248039
Да нет.
Аноним 15/01/16 Птн 19:35:07 #502 №248598 
>>248257
Схоронил пикчу.
sageАноним 15/01/16 Птн 19:35:24 #503 №248599 
>>248592
Пиздуй нахуй. ИТЕР ещё не построили даже. Зарепортил.
Аноним 15/01/16 Птн 19:42:52 #504 №248611 
14528761725750.jpg
>>248592
> Те же ИТЕР
Вот так он выглядел неделю назад, из пьезозажигалки ты получишь больше энергии чем с него. И тебе точно не в спейсач.
Аноним 15/01/16 Птн 20:13:19 #505 №248668 
переезд
https://2ch.hk/spc/res/248666.html
https://2ch.hk/spc/res/248666.html
https://2ch.hk/spc/res/248666.html
Аноним 16/01/16 Суб 15:14:16 #506 №249071 
14529464567330.jpg
14529464567471.jpg
первая фотка - галактика андромеды, вторая фотка - тоже галактика андромеды, только приближенная в нное количество раз. Яркая звезда на второй фотке - одна из видимых точек на первой фотки. Вопрос - что это за фигня вокруг на второй фотке? Тоже бесчисленные маленькие звезды? или пыль или что??
Аноним 17/01/16 Вск 12:12:00 #507 №249462 
>>249071
Звёзды. В этой галактике порядки триллиона звёзд. В нашей, для сравнения, в три раза меньше. Но всё равно оче дохуя.
comments powered by Disqus

Отзывы и предложения